Download as pdf or txt
Download as pdf or txt
You are on page 1of 48

Page 1 of 48

PTS 2021| All India Open Simulator Test 0 - Solutions | ForumIAS

Q.1)
Ans) c
Exp) Option c is correct.
Statement 1 is correct. Jal Jeevan mission is envisioned to provide safe and adequate drinking water through
individual household tap connection by 2024 to all households in rural India.
Statement 2 is incorrect. Ministry of Jal Shakti (not Ministry of rural development) is the nodal ministry
for the implementation of Jal Jeevan mission.
Statement 3 is correct. SDG 6 is about “clean water and sanitation for all”. Ensuring safe and adequate
drinking water is one of the objectives of the mission thus it is in accordance with SDG-6.
Statement 4 is correct. In Goa all rural households and in Telangana all rural households have already gotten a
FHTC by April 2021 (JJM dashboard).
Knowledge Base: The Service level delivery under JJM is to be 55 litres per capita per day.
In order to instill the ‘sense of ownership’ among the community/ user groups for better implementation and
long-term operation & maintenance of the scheme as well as bringing in transparency, GP/ VWSC/ Paani
Samiti will implement the in-village piped water supply infrastructure and related source development.
Communities will contribute towards 10% of the capital cost in cash and/or kind and/or labour in all villages
except for hilly and forested areas/ NE and Himalayan States and villages having more than 50% SC and/or ST
population, where community contribution would be 5% of the capital cost.
For targeted delivery and monitoring of specific outcomes, every functional tap connection is to be linked with
the Aadhar number of the head of the household subject to statutory provisions. Every asset created under JJM
will be geo-tagged. States will carry out inspections by empanelled third party agencies for all infrastructures
created under the JJM.
Source: https://jaljeevanmission.gov.in/
https://jalshakti-ddws.gov.in/sites/default/files/JJM_note.pdf
https://ejalshakti.gov.in/jjmreport/JJMIndia.aspx
https://www.livemint.com/news/india/govt-deploys-sensor-based-iot-devices-to-monitor-rural-drinking-
water-11617202052466.html
https://jaljeevanmission.gov.in/content/about-jjm#components

Q.2)
Ans) b
Exp) Option b is correct.
Business Advisory Committee: This committee regulates the business i.e., the programme of activities,
officially known as business of the House. For e.g. it decides which motions to be allotted time and when. The
business includes legislative and other types of business as well (administrative, etc.).
Strength in Lok Sabha - 15 (Committee Chairman - Speaker)
Strength in Rajya Sabha - (Committee Chairman - Chairman (RS))
Knowledge Base: Rules Committee
Aim - decide on the rules for procedures and conduct of business in the House.
Number of members - Lok Sabha: 15; Rajya Sabha: 16
Ex Officio Chairman - Lok Sabha: Speaker; Rajya Sabha: Chairman
General Purpose Committee

ForumIAS Guidance Center


2nd Floor, IAPL House, 19, Pusa Road, Karol Bagh, New Delhi – 110005 | helpdesk@forumias.academy| 9821711605
Page 2 of 48

PTS 2021| All India Open Simulator Test 0 - Solutions | ForumIAS

Aim - considers and advise on miscellaneous matters which do not fall into the clear cut jurisdiction of any
other Parliamentary Committee.
Number of members - Presiding Officer, Deputy Speaker/ Chairman, members of panel of Chairpersons (LS)
and Vice Chairpersons (RS), chairpersons of all departmentally related standing committees, leaders of
recognized parties/ groups within the House, members nominated by the Presiding Officer.
Ex Officio Chairman - Lok Sabha: Speaker; Rajya Sabha: Chairman
House Committee
Aim - managing the residential accommodation of members of House and other associated amenities e.g., food,
medical services, etc.
Number of members - Lok Sabha: 12; Rajya Sabha: Variable
Ex Officio Chairman - Elected from among members of respective Committee.
Source: Indian Polity by Laxmikanth (5th edition), Ch 23 Parliamentary Committees

Q.3)
Ans) d
Exp) Option d is correct.
Statement 1 is incorrect. Epidemic Diseases Act has been used time and again in order to deal with infectious
disease outbreaks like those of swine flu, dengue, malaria, etc. It was recently used in the swine flu outbreak of
2009 in Pune to set up screening centres and make swine flu notifiable disease, in Chandigarh to tackle malaria
and dengue in 2015.
Statement 2 is correct. The Epidemic Diseases Act, 1897 was amended in September 2020 in order to provide
for the safeguard of life and property of healthcare personnel and the premises and property of hospital and
healthcare service providers from violence during COVID 19 pandemic.
Statement 3 is correct. By invoking Section 2 of the Epidemic Diseases Act, advisories and directions of the
Ministry of Health & Family Welfare will be enforceable. It allows the government to inspect any ship arriving
or leaving any post and the power to detain any person intending to sail or arriving in the country.
Statement 4 is correct. The limitations on public gathering sizes, the curfews, restriction on transportation and
movement, mandatory wearing of masks in public places has all been ordered under the authority of this law.
Knowledge Base: Some important provisions of the Epidemic Diseases Act, 1897 are:
Section 2 - By invoking Section 2 of the Epidemic Diseases Act, advisories and directions of the Ministry of
Health & Family Welfare will be enforceable. It allows the government to inspect any ship arriving or leaving
any post and the power to detain any person intending to sail or arriving in the country.
Section 3 - provides penalties for disobeying any regulation or order made under the Act. These are according to
section 188 of the Indian Penal Code (Disobedience to order duly promulgated by a public servant)
Section 4 - gives legal protection to the implementing officers acting under the Act.
Amendments -
1) commission or abetment of acts of violence against healthcare service personnel shall be punished with
imprisonment for a term of three months to five years, and with fine of Rs 50,000 to Rs 2 lakh.
2) In case of causing grievous hurt, imprisonment shall be for a term of six months to seven years and a fine of
Rs1 lakh to Rs 5 lakh.
Source: https://indianexpress.com/article/explained/explained-what-is-the-epidemic-act-of-1897-govt-has-
invoked-to-fight-coronavirus-6309925/

ForumIAS Guidance Center


2nd Floor, IAPL House, 19, Pusa Road, Karol Bagh, New Delhi – 110005 | helpdesk@forumias.academy| 9821711605
Page 3 of 48

PTS 2021| All India Open Simulator Test 0 - Solutions | ForumIAS

Q.4)
Ans) b
Exp) Option b is correct.
Option a is correct. Special Leave Petition is a discretionary power of the Supreme Court. Hence, it cannot be
claimed as a matter of right by anyone. It can be granted in any judgement whether final or interlocutory.
Option b is incorrect. Special Leave Petition (SLP) has been mentioned in the constitution under the section
on Appellate powers of the Supreme Court under Article 136, Part V.
Option c is correct. The Supreme Court, while hearing a civil suit in the Cauvery dispute, had said that the
decision of the tribunal could be challenged before it through a Special Leave Petition under Article 136 of the
Constitution.
Option d is correct. Article 136 is fluid and flexible what it means that judgements, decrees or orders do not
have to be final in nature and appeals are allowed even against interlocutory and interim judgements.
Knowledge Base:
The Indian Judiciary is Integrated with a hierarchical structure of courts. Thus, the appeal from a lower court
generally lies first to its immediately higher court.
However, the Constitution of India under Article 136, vests the Supreme Court of India, with a special power to
grant special leave, to appeal against any judgment or order or decree in any matter or cause, passed or made by
any Court/Tribunal in the territory of India, except for military tribunal or court martial.
It is discretionary power vested in the Supreme Court of India and the court may in its discretion refuse to
grant leave to appeal.
SLP can be filed against any judgment of High Court within 90 days from the date of judgement; or SLP can be
filed within 60 days against the order of the High Court refusing to grant the certificate of fitness for appeal to
the Supreme Court. Any aggrieved party can file SLP against the judgment or order of refusal of grant of
certificate.
This petition is required to state all the facts that are necessary to enable the court to determine whether SLP
ought to be granted or not. It is required to be signed by Advocate on record. The petition should also contain a
statement that the petitioner has not filed any other petition in the High court. It should be accompanied by a
certified copy of judgement appealed against and an affidavit by the petitioner verifying the same and should
also be accompanied by all the documents that formed part of pleading in Lower court.
The appeal can be made in a case where a substantial question of law is involved or where gross injustice has
been observed. The judgment, decree or order against which the appeal is being made must have the character of
judicial adjudication. This implies that purely administrative or executive order or ruling cannot be a matter of
appeal and further, it is also important that the authority whose judgment or order is being appealed against
must fall under the definition of a court or a tribunal.
Source: Indian Polity by Laxmikanth (5th Edition), Ch 26 Supreme Court.

Q.5)
Ans) d
Exp) Option d is correct.
Option a is correct. Under article 348, the Constitution had originally provided that unless the Parliament so
provides all proceedings in the Supreme Court and high courts be conducted in English only. The Official
Languages Act, 1963 amended the constitution to allow the Governor of a state with previous consent of the

ForumIAS Guidance Center


2nd Floor, IAPL House, 19, Pusa Road, Karol Bagh, New Delhi – 110005 | helpdesk@forumias.academy| 9821711605
Page 4 of 48

PTS 2021| All India Open Simulator Test 0 - Solutions | ForumIAS

President to authorise the use of Hindi or any other official language of a state in the judgements, decrees, or
orders passed by state High Court along with the English translation. However, the Parliament has not made
any provision for the use of Hindi in the Supreme Court. Hence, the Supreme Court hears only those who
petition or appeal in English.
Option b and c is correct. Under Articles 345 and 346, regarding the Official Language of a state and the
language of official communication between states, the constitution has stipulated that:
1) a state legislature may adopt Hindi or any other (1 or more languages) spoken in the state as its official
language. Until then, English may be used for this purpose.
2) The link language for Union and non-Hindi states may be English, and for that between a Hindi and non-
Hindi state may be Hindi accompanied by English or just English only.
3) They can choose any language as its official language, including even those not mentioned in the 8th
Schedule of the Constitution.
Option d is incorrect. Under the section of Special Directives regarding Languages the Constitution provides
under Article 350 that an aggrieved person can draft a petition for redressal of grievance to any officer or
authority of the Union or a state, in any language preferred by him/ her even if it is in his mother tongue and not
in the Official Language. This has been done to ensure the protection of Linguistic Minorities and ensure that
they face no disablement or bias because of their mother tongue.
Source: Indian Polity by Laxmikanth (5th edition), Ch 61 Official Languages.

Q.6)
Ans) c
Exp) Option c is correct.
Option a is correct. Earlier if a beneficiary were to shift to another state, new ration cards has to be applied in
the second state. Now under one nation one ration card one ration card will be applicable nationwide. Thus,
beneficiaries from one state can get their ration from another state.
Option b is correct. One nation one ration card will also give the beneficiaries the opportunity to opt for the
dealer of their choice. If any dealer misbehaves or misallocates, the beneficiary can switch to another FPS shop
instantly.
Option c is incorrect. Ministry of Consumer Affairs, Food and Public distribution is the nodal ministry
under the scheme.
Option d is correct. There is no need to share or carry the ration card or Aadhaar card with ration dealer to
avail the benefit. Beneficiaries can undergo Aadhaar authentication by using their finger prints or iris based
identification.
Source: https://pib.gov.in/PressReleaseIframePage.aspx?PRID=1639584

Q.7)
Ans) a
Exp) Option a is correct
Universal adult suffrage means that all Indian citizens registered as voters have the right to cast their vote in
elections in order to choose their representatives. This is irrespective of their race, religion, caste and sex. This
right has been constitutionally guaranteed to all Indian citizens under Article 326. Thus, this right removes any
arbitrary distinctions between any two citizens based on wealth, or property or tax contribution, or education

ForumIAS Guidance Center


2nd Floor, IAPL House, 19, Pusa Road, Karol Bagh, New Delhi – 110005 | helpdesk@forumias.academy| 9821711605
Page 5 of 48

PTS 2021| All India Open Simulator Test 0 - Solutions | ForumIAS

level or membership of any identity like caste, gender or a particular religion, in the political sphere. Therefore,
this provision embodies the spirit of Political Justice in India.
Knowledge Base: Our Constitution envisages India as a Sovereign Democratic Republic with Social, Economic
and Political Justice (as per the Preamble).
Economic Justice means banishment of poverty and difference in status and opportunities - but not by
expropriation of those who have but by creating more resources and opportunities and equitable distribution of
fruits of national progress. This is done by progressive taxation, poverty alleviation schemes, right to work, etc.
Social Justice means removal of social imbalance by harmonising rival interests of various sections of societies.
Affirmative action, e.g., reservation in public education and employment in favor of minorities or backward
classes, is an example of social justice in action.
Source: An Introduction to the Constitution of India (DD Basu), Ch 3 Philosophy of the Constitution

Q.8)
Ans) d
Exp) Option d is correct.
Statement 1 is incorrect. Article 34 just implies that martial law may be applied in any territory of India. It also
empowers the Parliament to indemnify the Acts done by a government servant or any other person during the
enforcement of martial law but only for maintenance of order (e.g., sentence passed, punishment given,
forfeiture ordered, etc). Such an Act of Indemnity cannot be challenged in courts for violation of fundamental
rights. So Martial Law can affect only the fundamental rights and not centre- state relations (as in Emergency).
Statement 2 is incorrect. It has been borrowed as a concept from English common law.
Statement 3 is correct. Article 34 in Part III of Constitution only mentions the restrictions on Fundamental
Rights during the enforcement of Martial Law in India. It does not define what all is included in martial law
and in what situations it may be applied. The term has not been defined anywhere in the Constitution nor any
provision expressly authorizes the executive to declare martial law.
Source: Indian Polity by Laxmikanth (5th Edition), Ch 7 Fundamental Rights.

Q.9)
Ans) d
Exp) Option d is correct.
Statement 1 is correct: Under the Gram Nyayalayas Act 2008, it is for the state government to establish Gram
Nyayalayas in consultation with respective High Courts.
Statement 2 is correct: Control over subordinate courts is the collective and individual responsibility of the
High Court as it is the head of the judiciary in the state and has got administrative control over the subordinate
courts in respect of certain matters.
Statement 3 is correct. The Gram Nyayalaya will have both civil and criminal jurisdiction in cases specified in
the First and Second Schedules appended to the Gram Nyayalayas Act, 2008.
Statement 4 is correct: The permanent Lok Adalat shall have no jurisdiction in respect of any matter relating to
an offence not compoundable under any law.
Source: Indian Polity by Laxmikanth, Ch 35 Subordinate Courts.

ForumIAS Guidance Center


2nd Floor, IAPL House, 19, Pusa Road, Karol Bagh, New Delhi – 110005 | helpdesk@forumias.academy| 9821711605
Page 6 of 48

PTS 2021| All India Open Simulator Test 0 - Solutions | ForumIAS

Q.10)
Ans) d
Exp) Option d is correct.
Pair 1 is correctly matched. Washington Convention - CITES (the Convention on International Trade in
Endangered Species of Wild Fauna and Flora, also known as the Washington Convention) is a multilateral
treaty to protect endangered plants and animals. It was drafted as a result of a resolution adopted in 1963 at a
meeting of members of the International Union for Conservation of Nature (IUCN). The convention was
opened for signature in 1973 and CITES entered into force on 1 July 1975. Its aim is to ensure that international
trade in specimens of wild animals and plants does not threaten the survival of the species in the wild
Pair 2 is correctly matched. Geneva Convention - It was a series of international diplomatic meetings that
produced a number of agreements, in particular the Humanitarian Law of Armed Conflicts, a group of
international laws for the humane treatment of wounded or captured military personnel, medical personnel and
non-military civilians during war or armed conflicts. The agreements originated in 1864 and were significantly
updated in 1949 after World War II.
Genevan businessman, Henry Dunant was witness to the aftermath of the Battle of Solferino, a gory battle in
the Second War of Italian Independence. It affected him so much that he ultimately along with the Red Cross
formed a committee to help those affected by war giving birth to this convention.
Pair 3 is correctly matched. Bonn Convention - The Convention on the Conservation of Migratory Species of
Wild Animals, also known as the Convention on Migratory Species (CMS) or the Bonn Convention, is an
international agreement that aims to conserve migratory species throughout their ranges.[1] The Agreement was
signed under the auspices of the United Nations Environment Programme and is concerned with conservation
of wildlife and habitats on a global scale. It was signed in 1979 in Bonn, Germany.
Pair 4 is correctly matched. Rio Convention relates to the following three conventions, which were agreed at
the Earth Summit held in Rio de Janeiro in June 1992.
UNFCCC- United Nations Framework Convention on Climate Change
CBD -Convention on Biological Diversity
UNCCD - United Nations Convention to Combat Desertification
Source: https://en.wikipedia.org/wiki/List_of_international_environmental_agreements

Q.11)
Ans) b
Exp) Option b is correct.
Option a is correct. A law so enacted applies only to those states which have passed the resolutions. However,
any other state may adopt it afterwards by passing a resolution to that effect in its legislature.
Option b is incorrect. The bill to give effect to such a law can be initiated in any house of the parliament. Such
a law can be amended or repealed only by the Parliament and not by the legislatures of the concerned states
Option c is correct. The effect of passing a resolution under the above provision is that the Parliament becomes
entitled to legislate with respect to a matter for which it has no power to make a law. On the other hand, the
state legislature ceases to have the power to make a law with respect to that matter.
Option d is correct. Wild Life (Protection) Act of 1972 was passed under this provision. Other important acts
passed under this provision are - Prize Competition Act, 1955, Water (Prevention and Control of Pollution)
Act, 1974, Urban Land (Ceiling and Regulation) Act, 1976, Transplantation of Human Organs Act, 1994.

ForumIAS Guidance Center


2nd Floor, IAPL House, 19, Pusa Road, Karol Bagh, New Delhi – 110005 | helpdesk@forumias.academy| 9821711605
Page 7 of 48

PTS 2021| All India Open Simulator Test 0 - Solutions | ForumIAS

Source: M Laxmikanth, Indian Polity, 6th edition, page no. 14.3

Q.12)
Ans) a
Exp) Option a is correct
Option a is correct. Article 15 - Prohibition of discrimination on grounds of religion, race, caste, sex or place of
birth.
Article 17 - Abolition of Untouchability.
Thus, it ensures social equality.
Option b is incorrect. Article 48A - directive principles on protection and improvement of the environment,
and the safeguarding of forests and wildlife.
Article 49 - directive principles on the protection of monuments and places of and objects of national
importance.
It has nothing to do with social equality and is for protection of environment, monuments etc.
Option c is incorrect. These are provisions related to elections to ensure Political Equality.
Article 325 - No person to be excluded from electoral rolls on grounds of religion, race, caste or sex.
Article 326 - Elections to the House of People and to state legislative assemblies on basis of universal adult
suffrage.
Option d is incorrect. Article 239 - 241 - Administration of Union Territories.
Nothing to do with social equality.

Q.13)
Ans) a
Exp) Option a is correct
Statement 1 is correct. In a system borrowed from British Law (since the times of East India Company), the
Government is liable to be sued for torts (civil wrongs) by its officials only in non-sovereign functions and not
in course of sovereign functions like administering justice, commandeering goods during war, etc. This was
reaffirmed by the Supreme Court in the Kasturilal Case, 1965 post-independence as well. However lately the
Supreme Court has been giving a restricted interpretation as to what is sovereign function and has been
awarding compensation to victims in many cases.
Statement 2 is incorrect. In order to provide Constitutional offices like those of the President and Governor
independence and protection, they have been granted certain immunities:
1) With regards to their official acts - both during and after their terms, except for when impeachment
proceedings have been authorised by the parliament.
2) With regards to personal acts of criminal nature, no proceedings may be started or arrest/ imprisonment
done during their term of office. This immunity does not extend beyond that.
3) With regards to personal acts - civil proceedings may be instituted against them during their term of office,
but only after giving 2 months advance notice.
Source: Indian Polity by Laxmikanth (5th edition), Ch 64 Rights and Liabilities of the Government

Q.14)
Ans) c
Exp) Option c is correct.

ForumIAS Guidance Center


2nd Floor, IAPL House, 19, Pusa Road, Karol Bagh, New Delhi – 110005 | helpdesk@forumias.academy| 9821711605
Page 8 of 48

PTS 2021| All India Open Simulator Test 0 - Solutions | ForumIAS

Arthur Zimmermann was State Secretary for Foreign Affairs of the German Empire from 22 November 1916
until his resignation on 6 August 1917. His name is associated with the Zimmermann Telegram during World
War I.
However, he was closely involved in plans to support rebellions in Ireland and in India and to assist the
Bolsheviks to undermine Tsarist Russia.
As part of the Zimmerman Plan, German foreign office assisted in establishment of Berlin Committee for
Independence in 1915 by Virendranath Chattopadhyay, Bhupendernath Dutta, Lala Hardayal and others.
They aimed to mobilize Indian settlers abroad to send volunteers and arms to India to incite rebellion among
Indian troops.
Source: Spectrum Modern India 2015th edition 102 and
https://www.theworldwar.org/learn/wwi/zimmermann-telegram

Q.15)
Ans) a
Exp) Option a is correct.
Option a is correct. Pith denotes the ‘essence of something’ or the ‘true nature’, while substance states ‘the
most significant or essential part of something’. The Doctrine of Pith and Substance places emphasis on the
fact that it is the real subject matter which is to be challenged and not its incidental effects on another field.
It denotes the true nature of law. The power granted to legislatures to formulate a statute under three lists of the
seventh schedule in the Constitution of India is bound to overlap at certain points but this can’t be used as a
reason to make the whole statute null and void. Therefore, incidental effects or encroachments are permissible
under Constitutional Law while determining the competence of particular legislatures to the extent of subject
matters in the three lists is in question.
Option b is incorrect. As per the Doctrine of Colourable Legislation, if anything is prohibited directly then it
is also prohibited indirectly. Under the colour or guise of power given for one particular purpose, the legislature
cannot seek to achieve some other purpose which it is otherwise not competent to legislate on. If a subject
matter is in the State List, then Union cannot legislate upon such matter; and if a subject matter is in the Union
List, then State cannot legislate upon such matter.
Option c is incorrect. The Doctrine of Basic Structure signifies the basic features of the Constitution, which
cannot be amended, as they form the foundation of the Constitution on which its core principles stand.
Option d is incorrect. The Doctrine of Repugnancy deals with conflict between two pieces of legislation
which when applied to the same facts produce different results. Repugnancy arises when the provisions of two
laws are so inconsistent and irreconcilable that it is impossible to do one without disobeying the other. Article
254 of the Indian Constitution firmly entrenches the Doctrine of Repugnancy in India. This doctrine deals with
the conflict of law arises between Center and States. Article 254(1) states that if any provision of law or law
made by the legislature of the State is repugnant to the any provision of law or law made by the Parliament, then
the law made by Parliament will prevail over State enacted Law.
Source: https://flaviantnetwork-
my.sharepoint.com/personal/deepannita_flaviant_com/_layouts/15/onedrive.aspx?id=%2Fpersonal%2Fdeepa
nnita%5Fflaviant%5Fcom%2FDocuments%2FAttachments%2F27%2D49%2Epdf&parent=%2Fpersonal%2Fdee
pannita%5Fflaviant%5Fcom%2FDocuments%2FAttachments&originalPath=aHR0cHM6Ly9mbGF2aWFudG5
ldHdvcmstbXkuc2hhcmVwb2ludC5jb20vcGVyc29uYWwvZGVlcGFubml0YV9mbGF2aWFudF9jb20vX2xh

ForumIAS Guidance Center


2nd Floor, IAPL House, 19, Pusa Road, Karol Bagh, New Delhi – 110005 | helpdesk@forumias.academy| 9821711605
Page 9 of 48

PTS 2021| All India Open Simulator Test 0 - Solutions | ForumIAS

eW91dHMvMTUvZ3Vlc3RhY2Nlc3MuYXNweD9kb2NpZD0wM2EzODRjZTdhZmMyNDUyYmEzMTNi
YjdmOTYwOWRiMWMmYXV0aGtleT1BWFhCNkJyZDd3akFsLVZCQ0FEZEpXZyZydGltZT0wbWE1N
kl6cjJFZw

Q.16)
Ans) b
Exp) Option b is correct.
Option a is correct. The changes to Preamble (made only once so far) by the 42nd Constitution Amendment
Act, 1976 which introduced the words Socialist, Secular and Integrity to the Constitution.
Option b is incorrect. November 26, 1949 is the date mentioned in the preamble is the date on which people of
India in Constituent assembly adopted, enacted and gave to themselves this constitution (not enforced).
Option c is correct. Whether the Preamble is a part of the Indian constitution has been a topic of debate in
Indian jurisprudence. In the Berubari Union case, 1960, the Supreme Court had ruled that the Preamble is not a
part of the Constitution, although its contents may be used to understand the ideals that the makers of the
constitution wanted to base it on. However, in 1973 during the landmark Kesavananda Bharati case, the Supreme
Court revised its earlier stand and ruled that the Preamble is an integral part of the constitution.
Option d is correct. The Preamble closely resembles the contents of the Objectives Resolution drafted and
moved by Jawaharlal Nehru at the start of the work of the Constituent Assembly and represented the driving
ideals of the nation at the time.
Knowledge Base:
Also, to be noted is the fact that the Preamble is neither a source of power to the legislature nor a prohibition on
its powers and it is non-justiciable, that is its provisions are not enforceable in courts of law.
Source: Indian Polity by Laxmikanth (5th edition), Ch 4 Preamble of the Constitution

Q.17)
Ans) c
Exp) Option c is correct.
Option a is incorrect. Bond yield is the return an investor gets on that bond or on a particular government
security.
Option b is incorrect. The major factors affecting the yield is the monetary policy of the Reserve Bank of India,
especially the course of interest rates, the fiscal position of the government and its borrowing programme,
global markets, economy, and inflation.
Option c is correct. A rise in interest rates cause bond prices to fall, and bond yields to rise. Whereas a fall in
interest rates makes bond prices rise, and bond yields fall. In short, a rise in bond yields means interest rates in
the monetary system have fallen, and the returns for investors (those who invested in bonds and govt securities)
have declined.
Option d is incorrect. When bond yields go up, investors start reallocating investments away from equities and
into bonds, as they are much safer. As bond yields rise, the opportunity cost of investing in equities goes up, and
equities become less attractive.
Source: https://indianexpress.com/article/explained/explained-what-rise-in-bond-yield-means-for-investors-
and-govt-7205054/

ForumIAS Guidance Center


2nd Floor, IAPL House, 19, Pusa Road, Karol Bagh, New Delhi – 110005 | helpdesk@forumias.academy| 9821711605
Page 10 of 48

PTS 2021| All India Open Simulator Test 0 - Solutions | ForumIAS

Q.18)
Ans) a
Exp) Option a is correct
Statement 1 is incorrect. It is a village assembly consisting of persons registered in the electoral rolls (and
not all persons) of a village comprised within the area of panchayat at the village level.
Statement 2 is incorrect. If a panchayat is dissolved before its full duration, a new panchayat is elected within 6
months, but it functions only for the remainder of the period for which it dissolved panchayat would have
existed had it not been dissolved.
Statement 3 in correct. Unlike in parliament and state legislature elections, the candidates to panchayat
elections are qualified to do so when they become 21 years of age.
Source: Indian Polity by Laxmikanth (5th edition), Ch 38 Panchayati Raj,

Q.19)
Ans) d
Exp) Option d is correct.
Statement 1 is incorrect. Generally, there are two sets of electoral rolls - one common one for General and
state elections and a separate one for local body elections. The reason for existence of separate electoral rolls is
that the general and state elections are conducted by the Election Commission of India, while the local body
elections are conducted by respective State Election Commission. However, this practice is not rigidly or
uniformly followed throughout the country. Currently, all states, except Uttar Pradesh, Uttarakhand, Odisha,
Assam, Madhya Pradesh, Kerala, Odisha, Assam, Arunachal Pradesh, Nagaland and the Union Territory of
Jammu and Kashmir, adopt EC’s rolls for local body polls.
Statement 2 is incorrect. The Electoral Registration Officer (not Returning officer) is responsible for the
preparation of the electoral roll for each constituency.
Article 329 of the constitution lays down that no election to the Parliament or the state legislature is to be
questioned except by an election petition presented to such an authority as provided by the appropriate
Knowledge Base: The Prime Minister’s Office in mid 2020 had held a meeting with representatives of the
Election Commission and the Law Ministry to discuss the possibility of having a common electoral roll for
elections to the panchayat, municipality, state assembly and the Lok Sabha. The Prime Minister has often
pitched for a single voters list.
Article 329 of the constitution lays down that no election to the Parliament or the state legislature is to be
questioned except by an election petition presented to such an authority as provided by the appropriate
legislature. Therefore since 1966, election petitions are triable only in the High Courts. However appellate
jurisdiction on this matter still lies with the Supreme Court.
Also, Article 323 B empowers the Parliament or state legislature concerned to establish a tribunal for the
adjudication of an election dispute. However, no such tribunal has been established till date.
Source: https://indianexpress.com/article/explained/india-common-electoral-role-elections-voters-lists-
explained-6574403/
Indian Polity by Laxmikanth (5th Edition), Ch 68 Elections

Q.20)
Ans) c
Exp) Option c is correct.

ForumIAS Guidance Center


2nd Floor, IAPL House, 19, Pusa Road, Karol Bagh, New Delhi – 110005 | helpdesk@forumias.academy| 9821711605
Page 11 of 48

PTS 2021| All India Open Simulator Test 0 - Solutions | ForumIAS

Statement 1 is correct. Dissolution ends the very life of the existing House and necessitates that a new House be
formed after a general election. When the Lok Sabha is dissolved all business (motions, resolutions, bills,
amendments, notices, petitions, etc) pending before it or its Committees lapse and thus have to be reintroduced
in the reconstituted new Lok Sabha. An exception is all the Government Assurances that are pending
examination before the Committee on Government Assurances - these do not lapse upon dissolution are
taken up again after reconstitution.
Statement 2 is incorrect. The following bill lapses upon dissolution of the Lok Sabha:
1) A bill passed by the Lok Sabha but pending in the Rajya Sabha lapses.
2) A bill pending in the Lok Sabha lapses (whether originating in the Lok Sabha or transmitted to it by the Rajya
Sabha)
Statement 3 is correct. The following bills do not lapse upon dissolution of the Lok Sabha:
1) A bill not passed by the two Houses due to disagreement and for which the President has notified a joint
sitting before the dissolution.
2) A bill pending in the Rajya Sabha but not passed by the Lok Sabha.
3) A bill passed by both the Houses but awaiting the President’s assent.
4) A bill passed by both the Houses but returned by the President for reconsideration by the Houses.
Source: Indian Polity by Laxmikanth (5th edition), Ch 22 Parliament

Q.21)
Ans) b
Exp) Option b is correct.
Option b is correct. The Butler Committee (1927) was set up to examine the nature of relationship between the
princely states and government. It gave the following recommendations—
1) Paramountcy must remain supreme and must fulfil its obligations, adopting and defining itself according to
the shifting necessities of time and progressive development of states.
2) States should not be handed over to an Indian Government in British India, responsible to an Indian
legislature, without the consent of states. Thus, “paramountcy” was left undefined and this hydra-headed
creature was left to feed on usage, Crown’s prerogative and the princes’ implied consent.
Option a is incorrect. Hartog Committee (1929): The Hartog Committee was set up to report on
development of education. Its main recommendations were as follows: Emphasis should be given to primary
education but there need be no hasty expansion or compulsion in education. Only deserving students should go
in for high school and intermediate stage, while average students should be diverted to vocational courses after
VIII standard. For improvements in standards of university education, admissions should be restricted.
Option c is incorrect. The Aitchison Committee on Public Services (1886), set up by Dufferin. It
recommended—
1) dropping of the terms ‘covenanted’ and ‘uncovenanted’.
2) classification of the civil service into Imperial Indian Civil Service (examination in England), Provincial
Civil Service (examination in India) and Subordinate Civil Service (examination in India).
3) raising the age limit to 23.
Option d is incorrect. All Parties Conference met in February 1928 and appointed a committee under the
chairmanship of Motilal Nehru to draft a constitution. This was the first major attempt by the Indians to draft a

ForumIAS Guidance Center


2nd Floor, IAPL House, 19, Pusa Road, Karol Bagh, New Delhi – 110005 | helpdesk@forumias.academy| 9821711605
Page 12 of 48

PTS 2021| All India Open Simulator Test 0 - Solutions | ForumIAS

constitutional framework for the country. The recommendations of the Nehru Committee were unanimous
except in one respect—while the majority favoured the “dominion status” as the basis of the Constitution.
Source: A Brief History of Modern India Chapter 34 The Indian States Pg392, 546, 570, 637

Q.22)
Ans) c
Exp) Option c is correct.
Statement 1 is incorrect. Anekantavada is the fundamental doctrine of Jainism emphasises that the ultimate
truth and reality is complex and has multiple-aspects. Hence, there exists non-absolutism, that means no
single, specific statement can describe the nature of existence and the absolute truth.
Statement 2 is correct.
Sectarian lists the following eight auspicious symbols (Ashtamangala) under Jainsim:
1) Swastika: It signifies peace and wellbeing of the humans.
2) Nandavarta: It is a large swastika with nine end points.
3) Bhadrasana: A throne which is said to be sanctified by the Jaina’s feet.
4) Shrivatsa (endless knot): A mark which is manifested on the chest of tirthankara image and signifies his
pure soul.
5) Darpana: The mirror which reflects the inner self.
6) Minayugala: A couple of fish which signifies the conquest over sexual urges.
7) Vardhamanaka: A shallow dish used as lamp which shows the increase in the wealth, due and merit.
8) Kalasha: A pot filled with pure water signifying water.
Statement 3 is correct. Pratikraman – It is a process or an exercise during which Jains repent for their sins
during their daily life, and remind themselves not to repeat them. The five types of Pratikaman include Devasi,
Rai Pakhi, Chaumasi and Samvatsari.
Source: Nitin Singhania Chapter 14 BUDDHISM AND JAINISM - Pg no 481, 83

Q.23)
Ans) c
Exp) Option c is correct.
Option a is incorrect. Surendranath Banerjee and Ananda Mohan Bose were the main architects of the
Indian National Association. It was the most important of pre-Congress association and aimed to “promote by
every legitimate means the political, intellectual and material advancement of people”.
Option b is incorrect. Bose met Hitler under the pseudo name, Orlando Mazzotta. With the help of Hitler, the
‘Freedom Army’ (Mukti Sena) was formed which consisted of all the prisoners of war of Indian origin
captured by Germany and Italy. Dresden, Germany was made the office of the Freedom Army. Bose came to be
called ‘Netaji’ by the people of Germany. He gave the famous slogan, ‘Jai Hind’ from the Free India Centre,
Germany
Option c is correct. The younger section regarded the idea of dominion status in the report as a step backward,
and the developments at the All-Parties Conference strengthened their criticism of the dominion status idea.
Nehru and Subhash Bose rejected the Congress’ modified goal and jointly set up the Independence for
India League.

ForumIAS Guidance Center


2nd Floor, IAPL House, 19, Pusa Road, Karol Bagh, New Delhi – 110005 | helpdesk@forumias.academy| 9821711605
Page 13 of 48

PTS 2021| All India Open Simulator Test 0 - Solutions | ForumIAS

Option d is incorrect. During the First World War, Rashbehari Bose was involved as one of the leading
figures of the Ghadr Revolution. As the plan for revolution did not succeed, Rashbehari Bose escaped to Japan
in 1915. Much later, he was to play an important part in the founding of the Indian National Army.
Source: Spectrum Chapter 23 Quit India Movement, Demand for Pakistan and the INA Pg no 247, 457, 318

Q.24)
Ans) c
Exp) Option c is correct.
Statement 1 is the earliest in the chronology.
Chalukya dynasty emerged as a strong power with its founder Pulikesin I (c. 535- 566 CE) fortifying a hill near
Badami. He declared independence from the Kadmabas. It is said that he conducted yagnas and performed the
asvameda sacrifice. The capital Badami was founded by Kirtivarman (566-597).
Statement 2 is on second position in the chronology.
The iconic Shore Temple of Pallavas at Mamallapuram (Mahabalipuram) was constructed during the reign
of Rajasimha (700-728). The temple comprises three shrines, where the prominent ones are dedicated to Siva
and Vishnu. The exterior wall of the shrine, dedicated to Vishnu, and the interior of the boundary wall are
elaborately carved and sculpted. In southern India, this is one amongst the earliest and most important structural
temples.
Statement 3 is third on third position in the chronology.
After Dandidurga’s death in 758 CE, his uncle (father’s brother) Krishna I ascended the throne. Krishna I (756–
775 CE) defeated the Gangas of Mysore. The most striking structure at Ellora is the creation of the
Monolithic Kailasanath Temple. The temple was hewn out of a single rock during the time of Krishna I
(756–775 CE) in the 8th century.
Statement 4 is on the last position in the chronology.
Rajaraja I (985–1014) built the great Chola empire and constructed the Brihadeshwara temple in Thanjavur,
thereby bringing glory and fame to his dynasty.
Source: Tamil Nadu Textbook Chapter 9 Cultural Development in South India pg 144

Q.25)
Ans) b
Exp) Option b is correct.
Option b is correct. The Varadhanas of Taneshwar and Kannauj were responsible for turning out the Hun
invaders from India in the late 6th century. The most powerful of their kings was Harshavardhana whose
empire comprised almost the whole of Northern India.
The silver coins of the Vardhanas had on one side the head of the king and on the other side, the figure of a
peacock. The dates on the coins of Harshavardhana are reckoned in a new era, which most probably began in
606 AD, the year of his coronation.

ForumIAS Guidance Center


2nd Floor, IAPL House, 19, Pusa Road, Karol Bagh, New Delhi – 110005 | helpdesk@forumias.academy| 9821711605
Page 14 of 48

PTS 2021| All India Open Simulator Test 0 - Solutions | ForumIAS

Head of the King and Peacock


Option a is incorrect. The Western Chalukyan dynasty (6th century AD) was founded by Pulakeshin I with its
capital at Badami in Karnataka. One side of the coin had image of a temple or a lion and legends. The other side
was left blank. The coins of Eastern Chalukyan dynasty had symbol of the boar at the centre and the other side
here also was left blank.
Option c is incorrect. The coins issued by the Rajput dynasties (11th–12th century AD) were mostly of gold,
copper or billon (an alloy of silver and copper) but very rarely silver. One type showed the ‘name of the king in
Sanskrit on one side and a goddess on the other side’.
Option d is incorrect. The coins issued by Pandyan dynasty were square shaped with an image of elephant in
the early period. Later, fish became a very important symbol in the coins. The gold and silver coins had
inscriptions in Sanskrit and copper coins in Tamil.
Source: Nitin Singhani 24 COINS IN ANCIENT AND MEDIEVAL INDIA pg no 676

Q.26)
Ans) b
Exp) Option b is correct.
Statement 1 is incorrect. The Ramosis, the hill tribes of the Western Ghats, had not reconciled to British rule
and the British pattern of administration. They rose under Chittur Singh in 1822 and plundered the country
around Satara, Maharashtra. Govind Guru helped the Bhils of south Rajasthan (Banswara, Sunth states) to
organise themselves to fight for a Bhil Raj by 1913.
Statement 2 is correct. Tribal people resented the policy of annexation. After the annexation of the Maratha
territories by the British, the Ramosis, who had been employed by the Maratha administration, lost their
means of livelihood. This led them to revolt against the colonial rulers.
Statement 3 is incorrect. Generally, the British followed a pacifist policy towards the Ramosis, and even
recruited some of them into the hill police.
Source: Spectrum Chapter 6 People’s Resistance Against British Before 1857 pg 158

Q.27)
Ans) b
Exp) Option b is correct.
Statement 1 is correct. The Lahore Congress of I929 authorized the Working Committee to launch a
programme civil disobedience including non-payment of taxes. It had also called upon all members of
legislatures to resign their seats. In mid-February, 1930, the Working Committee, meeting at Sabarmati Ashram,
invested Gandhiji with fill powers to launch the Civil Disobedience Movement at a time and place of his choice.
Statement 2 is incorrect. On March 2, 1930, Gandhi informed the viceroy of his plan of action. According to
this plan (few realised its significance when it was first announced), Gandhi, along with a band of seventy-eight
members of Sabarmati Ashram, was to march from his headquarters in Ahmedabad through the villages of
Gujarat for 240 miles. The historic march, marking the launch of the Civil Disobedience Movement, began
on March 12, and Gandhi broke the salt law by picking up a lump of salt at Dandi on April 6.
Statement 3 is incorrect. The stated objective of the movement was complete independence and not just
remedying two specific wrongs and a vaguely-worded swaraj. Complete independence was declared as the aim
of the Congress at Lahore session, 1929.

ForumIAS Guidance Center


2nd Floor, IAPL House, 19, Pusa Road, Karol Bagh, New Delhi – 110005 | helpdesk@forumias.academy| 9821711605
Page 15 of 48

PTS 2021| All India Open Simulator Test 0 - Solutions | ForumIAS

Statement 4 is correct. On January 25, 1931, Gandhi and all other members of the Congress Working
Committee (CWC) were released unconditionally. The CWC authorised Gandhi to initiate discussions with
the viceroy. As a result of these discussions, a pact was signed between the viceroy, representing the British
Indian Government, and Gandhi, representing the Indian people, in Delhi on February 14, 1931. Gandhi’s
decision to suspend the civil disobedience movement was agreed under the Gandhi-Irwin Pact.
Source: India’s Struggle For Independence CHAPTER 22. CIVIL DISOBEDIENCE— 1930-31 pg 262, 264 A
Brief History of Modern India- spectrum- pg 380

Q.28)
Ans) b
Exp) Option b is correct.
Option a is incorrect. Ashwini Kumar Dutta was the founder of Brojomohan college named after his father.
Dutta has written a number of books in Bengali on religion, philosophy, and patriotism like Bhaktiyoga,
Karmayoga and Prem.
Option b is correct. The drain theory was put forward by Dadabhai Naoroji in his book Poverty and
UnBritish Rule in India. The major components of this drain were salaries and pensions of civil and military
officials, interests on loans taken by the Indian Government from abroad, profits on foreign investment in
India, stores purchased in Britain for civil and military departments, payments to be
made for shipping, banking and insurance services which stunted the growth of Indian enterprise in these
services.
Option c is incorrect. V.D. Savarkar wrote a book ‘The Indian War of Independence, 1857’. Savarkar called the
revolt the first war of Indian independence. He said it was inspired by the lofty ideal of self-rule by Indians
through a nationalist upsurge.
Option d is incorrect. Upsurge of working-class trade unionism after the War; the revolutionaries wanted to
harness the revolutionary potential of the new emergent class for nationalist revolution. Novels and books such
as Bandi Jiwan by Sachin Sanyal and Pather Dabi by Sharatchandra Chatterjee (a government ban only
enhanced its popularity) were among major influence to revolutionaries.
Source: A Brief History of Modern India Spectrum Chapter 28 Economic Impact of British Rule in India– pg
no 548, 380

Q.29)
Ans) b
Exp) Option b is correct.
In eastern part of the Gupta empire a peculiar system of land measurement was common with units known as
adhavapa, dronavapa and kulyavapa. Regarding the epigraphic evidence there is a reference to the units of
measurements, like kulyavapa, dronavapa, adhavapa and pataka from Bengal, nivartana and hhumi from central
India, and to nivartana and padavarta from western India.
Source: Indian History- Krishna Reddy CHAPTER 7 THE GUPTAS AND THEIR SUCCESSORS (AD 300–
750) pg no

Q.30)
Ans) a
Exp) Option a is correct.

ForumIAS Guidance Center


2nd Floor, IAPL House, 19, Pusa Road, Karol Bagh, New Delhi – 110005 | helpdesk@forumias.academy| 9821711605
Page 16 of 48

PTS 2021| All India Open Simulator Test 0 - Solutions | ForumIAS

Option a is correct. Kalsi Inscription: Kalsi town is known for its heritage importance. It is a small town
located in between Chakrata and Dehradun on the banks of Yamuna river. The site of Ashoka’s inscriptions at
Kalsi is singularly unique as it is the only place in North India where the great Mauryan emperor has
inscribed the set of the 14 rock edicts.
The language of these edicts is Prakrit and the script is Brahmi. The Rock edict was written around 450 BC and
is made of quartz which is 8 ft broad and 10 ft long. These rock edicts reflect Asoka’s human approach in his
internal administration when he converted himself to Buddhism.
It also reflects the policies for the commitment to non-violence and restriction of war. The inscription also tells
about his life when he took the path of spirituality.
Option b is incorrect. Prayag-Prashasti: Prayag-Prashasti is the name given to the Allahabad Pillar. “Prayag”
means a meeting place for something or someone. Prayag is an ancient name of Allahabad as it is the meeting
place or ‘Sangam’ of rivers Ganga, Yamuna and the mythical Saraswati.
Option c is incorrect. Maski Inscription: Maski is a village and an archaeological site in Raichur district of
Karnataka. It lies on the bank of the Maski river which is a tributary of the Tungabhadra. The site has a minor
rock edict of Emperor Ashoka. It was the first edict of Emperor Ashoka that contained the name Ashoka in it
instead of ‘Devanampriya’ or ‘Piyadassi’. The inscription remains a dharma shasana, and tells people to follow
the tenets of Buddhism.
Option d is incorrect. Kanganahalli Inscriptions: Located in Gulbarga district of Karnataka, this site contains
a Mahastupa of 75 ft diameter along with various sculptural slabs. A prominent one being the portrait slab of
King Ashoka and his queen attended by two chauri bearers. To confirm that it is Ashoka, the sculptural slab also
contains a single line label “Rayo Ashoka” in Brahmi script of the Satavahana period.
Source: Nitin Singhania Chapter 1 INDIAN ARCHITECTURE, SCULPTURE AND POTTERY pg 78

Q.31)
Ans) c
Exp) Option c is correct.
Statement 1 is correct. Ramananda has been described as “the bridge between the Bhakti movement of the
South and the North”.
Statement 2 is incorrect. Madhvacharya was the chief proponent of Dvaitavada (dualism) School of
Vedanta philosophy. He named his philosophy as “Tatvavada”. As per him, there lies a fundamental difference
between Atman (individual soul) and the Brahman (ultimate reality or God Vishnu). He was a critic of Adi
Shankara’s Advaita Vedanta and Ramanuja’s Vishishtadvaita Vedanta.
Statement 3 is correct. Nimbarka was a vaishnava saint and a chief proponent of Dvait-advait philosophy,
i.e., duality and nonduality at the same time (dualistic non-dualism). He founded a community called Nimbarka
Sampradaya.
Source: Nitin Singhania Appendix-2 BHAKTI AND SUFI MOVEMENT Pg no 737

Q.32)
Ans) a
Exp) Option a is correct.
Option a is correct. The Peshawar Conspiracy Cases were a set of five cases which took place between 1922
and 1927 in the British Raj. The defendants in these cases had allegedly snuck into British-controlled India from

ForumIAS Guidance Center


2nd Floor, IAPL House, 19, Pusa Road, Karol Bagh, New Delhi – 110005 | helpdesk@forumias.academy| 9821711605
Page 17 of 48

PTS 2021| All India Open Simulator Test 0 - Solutions | ForumIAS

the Soviet Union "to foment a proletarian revolution against the British imperialist oppressors and restore
freedom to the masses." The British Imperial government in the Indian subcontinent was terrified by the idea
of the spread of communism in the Raj which would bring freedom to the natives and subvert British interests
in India.
Option b is incorrect. Meerut Conspiracy Case: In March 1929, the Government arrested 31 labour leaders,
and the three-and-a-half-year trial resulted in the conviction of Muzaffar Ahmed, S.A. Dange, Joglekar, Philip
Spratt, Ben Bradley, Shaukat Usmani and others. They were arrested on the charge of conspiracy to overthrow
the British Government of India through strikes and other militant methods.
Option c is incorrect. Kakori Conspiracy Case: On 9 August 1925, ten men held up the 8-Down train at Kakori,
an obscure village near Lucknow, and looted its official railway cash. The Government reaction was quick and
hard. It arrested a large number of young men and tried them in the Kakori Conspiracy Case. Ashfaqulla Khan,
Ramprasad Bismil, Ràshan Singh and Rajendra Lahiri were hanged, four others were sent to the Andamans for
life and seventeen others were sentenced to long terms of imprisonment.
Option d is incorrect. Delhi-Lahore Conspiracy: It was organized in 1912 by the Indian revolutionary
underground in Bengal and Punjab and headed by Ras Bihari Bose to assassinate the then Viceroy of India, Lord
Hardinge. Basant Kumar Biswas, Amir Chand, and Avadh Behari were convicted and executed in the trial of
this Delhi Conspiracy Case. Source: Spectrum The Movement of the Working Class Peshawar Conspiracy
Cases - Wikipedia

Q.33)
Ans) c
Exp) Option c is correct.
Statement 1 is incorrect. In August 1921, peasant discontent erupted in the Malabar district of Kerala. Here
Mappila (Muslim) tenants rebelled. Their grievances related to lack of any security of tenure, renewal fees, high
rents, and other oppressive landlord exactions. It was a peasant movement which later transformed into a
communal movement and it was not a tribal movement.
Statement 2 is incorrect. Malabar rebellion also known as the Moplah rebellion was an armed revolt by the
Mappila Muslims of Kerala in 1921. It was started as a resistance against the British colonial rule, the prevailing
feudal system and in favour of the Khilafat Movement but ended in communal violence between Hindus and
Muslims.
Statement 3 is correct. Wagon Tragedy: A noteworthy event of the British suppression was the wagon tragedy
when approximately 60 Mappila prisoners on their way to prison, were suffocated to death in a closed railway
goods wagon.
Statement 4 is correct. Main Leaders of the Rebellion were Variyankunnath Kunjahammed Haji, Sithi Koya
Thangal and Ali Musliyar.
Source: ‘Moplah rioters’ not freedom fighters: report -ForumIAS Blog Bipin Chandra Peasant Movements and
Nationalism in the 1920s pg no 187

Q.34)
Ans) b
Exp) Option b is correct.
The Battle of Plassey: The officials of the Company made rampant misuse of its trade privileges that adversely
affected the nawab’s finances. The English fortified Calcutta without the nawab’s permission.

ForumIAS Guidance Center


2nd Floor, IAPL House, 19, Pusa Road, Karol Bagh, New Delhi – 110005 | helpdesk@forumias.academy| 9821711605
Page 18 of 48

PTS 2021| All India Open Simulator Test 0 - Solutions | ForumIAS

The Company further tried to mislead him, and compounded their sin by giving asylum to a political fugitive,
Krishna Das, son of Raj Ballabh who had fled with immense treasures against the nawab’s will. The Company,
on its part, suspected that Siraj would drastically reduce its trade privileges in collusion with the French in
Bengal. Thus, when Siraj attacked and seized the English fort at Calcutta, it brought their hostility into the open.
Mention may be made here of the much propagated ‘Black Hole Tragedy’. Siraj-ud-daula is believed to have
imprisoned 146 English persons who were lodged in a very tiny room due to which 123 of them died of
suffocation. However, historians either do not believe this story, or say that the number of victims must have
been much smaller.
Source: Spectrum’s A Brief History of Modern India, Page 88.

Q.35)
Ans) a
Exp) Option a is correct.
During Swadeshi Movement, picketing before the shops selling British goods led to a boycott of the officially
controlled educational institutions. The British threat to the student-picketers in the form of the withdrawal
of grants, scholarships and affiliations of the institutions to which they belonged through the infamous
circular of 22 October, 1905 issued by Carlyle, the Chief Secretary of the Government of Bengal, known
otherwise as the "Carlyle Circular".
The chief secretary to the government of Bengal, RW Carlyle, sent a secret circular on October 10, asking
government-funded schools and colleges to prohibit students from participating in the protest.
Cunningham Circular of 1930: The circular was aimed to forbade students from participating in political
activities and so, it raised a very strong response.
Source: Unit-11.pdf (egyankosh.ac.in) Spectrum pg no 374

Q.36)
Ans) c
Exp) Option c is correct.
Statement 1 is incorrect. Tamil has two major texts written in 6th century AD, i.e., Silappadikaram (tale of an
anklet) which was written by Ilango-Adigal. The second text is Manimekalai (the story of Manimekalai) written
by Satthanar. These texts focus on the Tamil society and the economic and political changes it was experiencing.
Statement 2 is correct. Malayalam is usually spoken in Kerala and surrounding areas. Two of the major
Malayalam works of the medieval period are Kokasandisan and Bhasa Kautilya (a commentary on Arthashastra).
Another major literary work in Malayalam is Ramacharitam, an epic poem written by Cheeraman in 12th
century. Ezhuthachan, a strong proponent of Bhakti movement, is known as the Father of Malayalam
Language.
Statement 3 is incorrect. Krishnadevaraya (1509-1529), while travelling via Vijayawada during his Kalinga
campaign, himself wrote Amuktamalyada which is a treatise stating the instance of Lord Vishnu in his
dream. During Krishnadevaraya’s reign, eight learned Telugu poets were attached to his court. They were called
ashtadiggajas.
Tenali Ramakrishna (Court jester and poet). He is said to have been a duo with the king and the stories of Tenali
Raman have been circulated even in modern times. His work includes Panduranga Mahatmayam.
Source: Nitin Singhania Chapter 15 INDIAN LITERATURE Pg no 514

ForumIAS Guidance Center


2nd Floor, IAPL House, 19, Pusa Road, Karol Bagh, New Delhi – 110005 | helpdesk@forumias.academy| 9821711605
Page 19 of 48

PTS 2021| All India Open Simulator Test 0 - Solutions | ForumIAS

Q.37)
Ans) b
Exp) Option b is correct.
The University of Cambridge has found out new genes emerging in the genome of living organisms. They
have called them novel Open Reading Frames or as nORFs.
Human Genome Project (HGP) was an international, collaborative research program conducted from 1990 to
2003. The main goal was to completely map and understand all the genes of human beings.
But even after the research, the number of ‘known’ genes has remained constant for two decades. Hence, a team
from the University of Cambridge has set out to find whether new genes emerge in the genome of living
organisms and if they do, how they do so.
Novel Regions: These novel regions were explored and cannot be defined by our current ‘definition’ of a gene.
Hence, these novel regions are called novel Open Reading Frames or as nORFs. These regions are also broadly
involved in diseases. They were seen as dysregulated (poor ability to manage) in 22 cancer types.
Source: What are "novel Open Reading Frames(nORFs)"? ForumIAS Blog

Q.38)
Ans) d
Exp) Option d is correct.
Edge computing enables data to be analysed, processed and transferred at the edge of a network. The idea is to
analyse data locally closer to where it is stored, in real-time without latency rather than send it far away to a
centralised data centre.

Application of Edge Computing can be realised in the following:


1) Autonomous vehicles: With edge computing, it will be possible to remove the need for drivers in all trucks
except the front one, because the trucks will be able to communicate with each other with ultra-low latency.
2) Virtualised radio networks and 5G (vRAN): Operators are increasingly looking to virtualise parts of their
mobile networks (vRAN). This has both cost and flexibility benefits. The new virtualised RAN hardware
needs to do complex processing with a low latency. Operators will therefore need edge servers to support
virtualising their RAN close to the cell tower.

ForumIAS Guidance Center


2nd Floor, IAPL House, 19, Pusa Road, Karol Bagh, New Delhi – 110005 | helpdesk@forumias.academy| 9821711605
Page 20 of 48

PTS 2021| All India Open Simulator Test 0 - Solutions | ForumIAS

3) Traffic management: Edge computing can enable more effective city traffic management. Examples of this
include optimising bus frequency given fluctuations in demand, managing the opening and closing of extra
lanes, and, in future, managing autonomous car flows.
4) In-hospital patient monitoring: An edge on the hospital site could process data locally to maintain data
privacy. Edge also enables right-time notifications to practitioners of unusual patient trends or behaviours
(through analytics/AI), and creation of 360-degree view patient dashboards for full visibility.
5) Smart grid: Sensors and IoT devices connected to an edge platform in factories, plants and offices are being
used to monitor energy use and analyse their consumption in real-time. With real-time visibility, enterprises
and energy companies can strike new deals, for example where high-powered machinery is run during off-
peak times for electricity
6) demand. This can increase the amount of green energy (like wind power) an enterprise consumes.
7) Many such application can be realised through Edge Computing.

Q.39)
Ans) a
Exp) Option a is correct.
Statement 1 is correct. Somatic cell nuclear transfer involves transferring the nucleus of a cell, which includes
its DNA, into an egg which has had its nucleus removed.
Process:
A somatic cell is isolated and extracted from an adult female. Then the nucleus and all of its DNA from an egg
cell is removed. After that the nucleus from the somatic cell is transferred to the egg cell. After being inserted
into the egg, the somatic cell nucleus is reprogrammed by the host cell and is stimulated with a shock. The egg
cell, with its new nucleus, will behave just like a freshly fertilised egg. It developed into an embryo, which is
implanted into a surrogate mother.
Statement 2 is correct. Somatic Cell Nuclear Transfer technique is currently the basis for cloning animals
(such as the famous Dolly the sheep), and has been theoretically proposed as a possible way to clone
humans. But there are moral and ethical objections against reproductive cloning.
Statement 3 is correct. The application of Somatic Cell Nuclear Transfer research to generate tissues or even
organs for transplant into the specific patient. The resulting cells would be genetically identical to the somatic
cell donor, thus avoiding any complications from immune system rejection.
Statement 4 is incorrect. In 2018, Chinese Scientists have managed to clone two identical long-tailed
macaques, the first primates to have undergone cloning. This brings human cloning one step closer.
Source: [Answered] What is Somatic cell nuclear transfer? Explain its technology and enlist various potential
application of Somatic Cell Nuclear Transfer technology. -ForumIAS Blog
Chinese scientists successfully clone monkeys; humans could be next | Technology News,The Indian Express

Q.40)
Ans) b
Exp) Option b is correct.
Statement 1 is correct. Natural language processing (NLP) is a branch of artificial intelligence that deals with
the interaction between computers and humans using the natural language. The ultimate objective of NLP is to

ForumIAS Guidance Center


2nd Floor, IAPL House, 19, Pusa Road, Karol Bagh, New Delhi – 110005 | helpdesk@forumias.academy| 9821711605
Page 21 of 48

PTS 2021| All India Open Simulator Test 0 - Solutions | ForumIAS

read, decipher, understand, and make sense of the human languages in a manner that is valuable. Most NLP
techniques rely on machine learning to derive meaning from human languages.
Statement 2 is correct. NLP entails applying algorithms to identify and extract the natural language rules such
that the unstructured language data is converted into a form that computers can understand.
Statement 3 is incorrect. NLP can be used in sentiment analysis (understand mood of the customer), topic
modelling (discovering abstract topics that occur in a collection of documents), text categorisation (categorising
text into organised groups).
Natural Language Processing is the driving force behind Personal assistant applications such as OK Google, Siri,
Cortana, and Alexa.
Source: NLP has become one of the most exciting parts of AI (indiatimes.com)
A Simple Introduction to Natural Language Processing | by Dr. Michael J. Garbade | Becoming Human:
Artificial Intelligence Magazine

Q.41)
Ans) d
Exp) Option d is correct.
Statement 1 is incorrect. Bose-Einstein condensate (BEC) is a state of matter in which separate atoms or
subatomic particles, cooled to near absolute zero, coalesce into a single quantum mechanical entity. It can be
described by a wave function on a near-macroscopic scale. This form of matter was predicted in 1924 by Albert
Einstein on the basis of the quantum formulations of the Indian physicist Satyendra Nath Bose.
Statement 2 is correct. The most intriguing property of BECs is that they can slow down light. Speed of light
decreases when it passes through Bose Einstein Condensate. This feature hold promises for new types of light-
based telecommunications, optical storage of data, and quantum computing, though the low-temperature
requirements of BECs offer practical difficulties.
Statement 3 is correct. The matter waves of Bose-condensed atoms are coherent (laser like). In physics,
coherence is a fixed relationship between the phase of waves in a beam of radiation of a single frequency. Two
beams of light are coherent when the phase difference between their waves is constant.
Source: https://www.britannica.com/science/Bose-Einstein-condensate
https://physicsworld.com/a/bose-einstein-condensation/

Q.42)
Ans) d
Exp) Option d is correct.
Statement 1 is incorrect. Ransomware is a type of malicious software that infects a computer and restricts
users’ access to it until a ransom is paid to unlock it.
Statement 2 is correct. Ransomware is often spread through phishing emails that contain malicious
attachments or through drive-by downloading. Drive-by downloading occurs when a user unknowingly visits
an infected website and then malware is downloaded and installed without the user’s knowledge.
Statement 3 is correct. A ransomware typically logs users out of their own systems through forced encryption
of data and asks them to pay a ransom if they want to access the encrypted data.
Statement 4 is correct. In 2020, IT services companies in India were attacked by the 'Maze' ransomware which
caused disruptions to some of their clients.
Knowledge Base: https://security.berkeley.edu/faq/ransomware/

ForumIAS Guidance Center


2nd Floor, IAPL House, 19, Pusa Road, Karol Bagh, New Delhi – 110005 | helpdesk@forumias.academy| 9821711605
Page 22 of 48

PTS 2021| All India Open Simulator Test 0 - Solutions | ForumIAS

https://www.business-standard.com/article/companies/cognizant-hit-by-maze-ransomware-attack-says-
clients-face-disruption-120041900619_1.html
https://indianexpress.com/article/technology/tech-news-technology/petya-ransomware-cyberattack-
explained-hits-europe-what-it-does-how-to-protect-your-pc-and-more-4725476/
Q.43)
Ans) c
Exp) Option c is correct.
LUPEX is a robotic lunar mission by the Indian Space Research Organisation (ISRO) and Japan Aerospace
Exploration Agency (JAXA).
Aim: The mission aims to send a lunar rover and lander to explore the South Pole region of the Moon in 2024.
Objective: The mission will demonstrate new surface exploration technologies related to vehicular transport. It
will also test lunar night survival capabilities for sustainable lunar exploration in the South Pole region of the
moon.
Contribution: JAXA is likely to provide the H3 launch vehicle and the rover. While ISRO would be responsible
for the lander for this mission.
Source: https://blog.forumias.com/factly-news-articles-for-upsc-prelims-mar-12-2021/

Q.44)
Ans) a
Exp) Option a is correct.
Statement 1 is correct. Gene, is a unit of hereditary information that occupies a fixed position on a
chromosome. Genes are passed from parents to offspring and contain the information needed to specify traits.
Genes achieve their effects by directing the synthesis of proteins. A chromosome contains a single, long DNA
molecule, only a portion of which corresponds to a single gene.
Statement 2 is incorrect. Offspring receive 23 chromosomes each from their mother and their father. Sex
chromosomes received from the father determine the sex of the offspring.
Statement 3 is correct: Genes are segments of deoxyribonucleic acid (DNA) that contain the code for a
specific protein that functions in one or more types of cells in the body. Genes are made of DNA, which are a
double strands of deoxyribonucleic acid molecule.
Statement 4 is correct. The particles in the nucleus of the cell, responsible for heredity, are called chromosomes
which are made up of proteins and another type of biomolecules called nucleic acids.
Nucleic acids are responsible for the transfer of characters from parents to off springs. There are two types of
nucleic acids — DNA and RNA. A nucleic acid containing deoxyribose is called deoxyribonucleic acid (DNA)
while that which contains ribose is called ribonucleic acid (RNA).
Both DNA and RNA contain Adenine, Guanine and Cytosine. The fourth base is Thymine in DNA and Uracil
in RNA. The structure of DNA is a double strand helix whereas RNA is a single strand molecule.
Knowledge Base: Recombinant DNA (rDNA) molecules are DNA molecules formed by laboratory methods of
genetic recombination (such as molecular cloning) to bring together genetic material from multiple sources,
creating sequences that would not otherwise be found in the genome.
Recombinant DNA is possible because DNA molecules from all organisms share the same chemical structure.
They differ only in the nucleotide sequence within that identical overall structure.
Source: https://ncert.nic.in/textbook/pdf/lech205.pdf

ForumIAS Guidance Center


2nd Floor, IAPL House, 19, Pusa Road, Karol Bagh, New Delhi – 110005 | helpdesk@forumias.academy| 9821711605
Page 23 of 48

PTS 2021| All India Open Simulator Test 0 - Solutions | ForumIAS

Q.45)
Ans) d
Exp) Option d is correct.
Statement 1 is correct. Proton therapy, also called proton beam therapy, is a type of radiation therapy. It uses
protons rather than x-rays to treat cancer. A proton is a positively charged particle. At high energy, protons
can destroy cancer cells. Doctors may use this therapy alone. They may also combine it with x-ray radiation
therapy, surgery, chemotherapy, and/or immunotherapy.
Statement 2 is correct. In regular radiation therapy, the beam of energy goes into the body, through the tumor,
and out the other side. This “exit dose” of radiation might affect healthy tissue beyond the tumor. After
delivering the energy to the tumor, the protons stop: They do not exit the tumor and go into healthy tissue
on the other side. In this way, proton therapy reduces radiation exposure and potential damage to healthy
tissue, especially in sensitive areas such as the brain, eyes, spinal cord, heart, major blood vessels and nerves.
Statement 3 is correct. Proton, a stable subatomic particle that has a positive charge equal in magnitude to a
unit of electron charge and a rest mass of 1.67262 × 10−27 kg, which is 1,836 times the mass of an electron. When
the number of protons in a nucleus equals the number of electrons orbiting the nucleus, the atom is electrically
neutral.
Knowledge Base: Cancer is a disease in which abnormal cells divide uncontrollably and destroy body tissue.
Source: https://www.thehindu.com/news/cities/chennai/apollo-hospitals-inaugurates-proton-cancer-
centre/article26083995.ece
https://www.proton-therapy.org/science
https://www.hopkinsmedicine.org/health/treatment-tests-and-therapies/proton-therapy

Q.46)
Ans) d
Exp) Option d is correct.
Option a is incorrect. Click Wrap Signatures is used for mostly online purchases, where a simple tick in a box
is an acceptance of the terms and conditions contained; referred to somewhere on the website. A user is
required to click the box before the services can be consumed.
Option b is incorrect. Electronic Signature (E-Signature) is a digital version of a wet signature, where many
countries have defined the legality and description of uses for e-signatures. These are the most common form
of signatures used by organizations globally.
Option c is incorrect. Digital Signature is an authentication mechanism that enables a code to be attached as
a signature. It is required for certain specific agreements, and issued through a certification agency.
Option d is correct. The wet signature is any physical mark on documents created by a person. In most cases
its writing your name in cursive on a piece of paper, document or contract, often with initials on each page
indicating the extent of what is being signed.Source:
https://realyst.com/digital-transaction-management/different-types-of-signatures/

Q.47)
Ans) b
Exp) Option b is correct.

ForumIAS Guidance Center


2nd Floor, IAPL House, 19, Pusa Road, Karol Bagh, New Delhi – 110005 | helpdesk@forumias.academy| 9821711605
Page 24 of 48

PTS 2021| All India Open Simulator Test 0 - Solutions | ForumIAS

Statement 1 is incorrect. It is m-RNA vaccine which uses the genetic sequence of a portion of the virus that
can be injected into the body. Whereas in inactivated vaccines, the killed version of the germ is injected,
which cause a disease.
Statement 2 is correct. mRNA vaccines have several benefits compared to other types of vaccines which
includes shorter manufacturing times and, because they do not contain a live virus, no risk of causing disease
in the person getting vaccinated.
Statement 3 is incorrect. Live vaccines use a weakened (or attenuated) form of the germ that causes a disease.
They are similar to the natural infection that they help prevent, thus create a strong and long-lasting immune
response. Just 1 or 2 doses of most live vaccines can give you a lifetime of protection. It is inactivated vaccines
which usually don’t provide immunity as strong as live vaccines. So, you may need several booster shots in
order to get ongoing immunity against diseases.
Source: https://www.thehindu.com/news/national/coronavirus-indias-m-rna-vaccine-could-be-ready-in-
march/article33068719.ece and https://www.vaccines.gov/basics/types

Q.48)
Ans) d
Exp) Option d is correct.
E-Waste is a term used to cover items of all types of electrical and electronic equipment (EEE) and its parts
that have been discarded by the owner as waste without the intention of re-use.
Electronic waste is considered the fastest growing waste stream in the developed world. The waste is the result
of advancing technology that leaves behind old computers, laptops, televisions, and other electronic devices
that require specialized methods of recycling because of toxic by-products. A major concern regarding end-
of-life products (EOL) is environmental contamination and seepage into the food chain.
List of some common e-waste items:
1) Electric cookers
2) Heaters Laptops
3) Circuit boards
4) Hard Drives
5) Solar Panels
6) Power Distribution Systems (PDU’s)
7) Autoclave
8) Defibrillator
9) Smart Watches
10) Heart Monitors
11) Diabetic Testing EquipmentKnowledge Base:
12) Defibrillators are devices that restore a normal heartbeat by sending an electric pulse or shock to the heart.
They are used to prevent or correct an arrhythmia, a heartbeat that is uneven or that is too slow or too fast.
13) Solar modules use potentially hazardous materials, including lead compounds, polymers and cadmium
compounds. If disposed of in an inappropriate way, potential leaching of those hazardous materials can have
negative environmental and health impacts.
Source: https://www.ewaste1.com/what-is-e-waste/

ForumIAS Guidance Center


2nd Floor, IAPL House, 19, Pusa Road, Karol Bagh, New Delhi – 110005 | helpdesk@forumias.academy| 9821711605
Page 27 of 48

PTS 2021| All India Open Simulator Test 0 - Solutions | ForumIAS

Source: https://www.rbi.org.in/Scripts/FAQView.aspx?Id=92

Q.54)
Ans) a
Exp) Option a is correct.
Statement 1 is correct: Regulatory forbearance for banks involved relaxing the norms for restructuring
assets, where restructured assets were no longer required to be classified as Non-Performing Assets (NPAs
henceforth) and therefore did not require the levels of provisioning that NPAs attract.
Statement 2 is incorrect: Regulatory Forbearance is a regulatory policy (i.e., a policy implemented by central
banks and other regulatory authorities) that permits banks and financial institutions to continue operating even
when their capital is fully depleted. It involves relax norms and control of financial institutions by the
regulators.
Statement 3 is incorrect. It results into undercapitalization of Banks and not overcapitalization.
The P. J. Nayak Committee (2014), stated, “the existing tier-I capital for public sector banks is overstated
because of the regulatory forbearance which RBI provides on restructured assets. Without forbearance these
assets would be categorized as NPAs, the restructuring being a response to likely imminent default. As a
consequence, provisioning would rise, and tier-I capital would fall.” Thus, in essence, many banks were
undercapitalized during the forbearance period. The report had estimated that if regulatory forbearance were
withdrawn immediately in May 2014 and a prudent 70% provision cover were provided for restructured assets,
tier-1 capital of the public sector banks would be written down by INR 2.78 lakh crores.
Source: https://blog.forumias.com/wp-content/uploads/2021/02/Economic-Survey-2020-21-Vol-1-
Summary-7.pdf

Q.55)
Ans) a
Exp) Option a is correct.
Option a is incorrect. The One Sun One World One Grid project is initiated by India. The idea was first made
2018, by the Prime Minister during the first assembly of the International Solar Alliance (Headquartered in
Gurugram). It is being implemented by the Ministry of New and Renewable Energy, Government of India.
Option b is correct. The fundamental concept behind the project is to develop a trans-national grid supply of
solar power that will be laid all over the globe. Through this one country can sell solar power to other solar scarce
or cold countries.
Option c is correct. The OSOWOS project plan is to lay undersea cables for transferring solar energy to other
countries. Through this it plans to build a global ecosystem of interconnected renewable energy resources that
are seamlessly shared for mutual benefits and global sustainability.
Option d is correct. The project would have three phases. In first phase Phase I, Middle East, South Asia and
South East Asia would be interconnected. In the second phase, solar and other renewable energy resources rich
regions would be interconnected. In the third phase the project would aim for global interconnection of the
power transmission grid to achieve the One Sun One World One Grid vision.
Source: https://timesofindia.indiatimes.com/readersblog/random-things/one-sun-one-world-one-grid-25819/

ForumIAS Guidance Center


2nd Floor, IAPL House, 19, Pusa Road, Karol Bagh, New Delhi – 110005 | helpdesk@forumias.academy| 9821711605
Page 28 of 48

PTS 2021| All India Open Simulator Test 0 - Solutions | ForumIAS

https://energy.economictimes.indiatimes.com/news/renewable/mnre-begins-process-to-engage-consultancy-
for-one-sun-one-world-one-grid-plan/76082301

Q.56)
Ans) d
Exp) Option d is correct.
Statement 1 is incorrect. Organization of the Petroleum Exporting Countries (OPEC) is not an informal
forum, but is a permanent, intergovernmental Organization created at the Baghdad Conference in 1960 by Iran,
Iraq, Kuwait, Saudi Arabia and Venezuela.
Statement 2 is incorrect. The OPEC Fund for International Development is a multilateral development
finance institution established in 1976. The OPEC Fund is dedicated to improving people’s lives by providing
public, private, and trade sector financing, as well as grants, to support sustainable social and economic
advancement across the globe.
Statement 3 is incorrect. The non-OPEC countries which export crude oil are termed as OPEC plus.
Source: 4th High-Level Meeting of the OPEC-India Dialogue | | Factly (forumias.com)
OPEC: The OPEC Fund

Q.57)
Ans) a
Exp) Option a is correct.
Statement 1 is correct. Priority Sector lending (PSL) means those sectors which the Government and RBI
consider as important for the development of the basic needs of the country and are to be given priority over
other sectors. The banks are mandated to encourage the growth of such sectors with adequate and timely credit.
Statement 2 is incorrect. Commercial banks including foreign banks are required to mandatorily earmark 40%
of the adjusted net bank credit for priority sector lending. Regional rural banks and small finance banks will
have to allocate 75% of adjusted net bank credit to PSL.
Statement 3 is incorrect. The categories included in the Priority Sector lending (PSL) list are Agriculture b)
Micro, Small and Medium Enterprises c) Export Credit d) Education e) Housing f) Social Infrastructure g)
Renewable Energy and h) Others.
Real estate and consumer goods are not included in the categories for Priority Sector lending.
Source: Priority Sector Lending (PSL) guidelines -ForumIAS Blog

Q.58)
Ans) a
Exp) Option a is correct
Statement 1 is incorrect. When a country makes a conscious decision to lower the value of its currency in a
fixed exchange rate it is called devaluation (and not depreciation). Depreciation is, when there is a fall in the
value of a currency in a floating exchange rate. This is not due to a government’s decision, but due to supply
and demand-side factors.

ForumIAS Guidance Center


2nd Floor, IAPL House, 19, Pusa Road, Karol Bagh, New Delhi – 110005 | helpdesk@forumias.academy| 9821711605
Page 29 of 48

PTS 2021| All India Open Simulator Test 0 - Solutions | ForumIAS

Statement 2 is correct. Depreciation could cause higher economic growth. Part of aggregate demand is
(Export-Import) therefore higher exports and lower imports should increase aggregate demand (assuming
demand is relatively elastic).
Statement 3 is correct. J Curve is an economic theory which states that a country’s balance of trade initially
worsens following a devaluation of its currency, then quickly recovers and finally surpasses its previous
performance. It happens primarily because in the near-term, higher prices on imports will have a greater impact
on total nominal imports than the reduced volume of imports. J-curve effect is also seen in case of private
equity investments.

Source: https://www.economicshelp.org/blog/355/trade/devaluation-and-depreciation-
definition/#:~:text=In%20general%2C%20everyday%20use%2C%20devaluation,depreciation%20are%20often
%20used%20interchangeably.&text=Essentially%20devaluation%20is%20changing%20the,in%20a%20floating
%20exchange%20rate, https://www.economicshelp.org/macroeconomics/exchangerate/effects-devaluation/
https://www.economicshelp.org/blog/10050/economics/effects-appreciation/
https://www.investopedia.com/terms/j/j-curve-
effect.asp#:~:text=The%20J%2Dcurve%20effect%20is,finally%20surpasses%20its%20previous%20performanc
e

Q.59)
Ans) b
Exp) Option b is correct.
Statement 1 is correct. The payments bank will be registered as a public limited company under the Companies
Act, 2013, and licensed under Section 22 of the Banking Regulation Act, 1949, with specific licensing conditions
restricting its activities mainly to acceptance of demand deposits and provision of payments and remittance
services.
Statement 2 is incorrect. The payments bank cannot undertake lending activities. It is allowed to issue ATM /
Debit Cards. Payments banks, however, cannot issue credit cards.
Statement 3 is correct. The payments bank is required to use the words “Payments Bank” in its name in order to
differentiate it from other banks.
Statement 4 is incorrect. The payments bank cannot set up subsidiaries to undertake non-banking financial
services activities. The other financial and non-financial services activities of the promoters, if any, should be
kept distinctly ring-fenced and not comingled with the banking and financial services business of the payments
bank.
Knowledge Base: What is a Payment Bank?
A payments bank is like any other bank, but operating on a smaller scale without involving any credit risk. In
simple words, it can carry out most banking operations but can’t advance loans or issue credit cards. It can

ForumIAS Guidance Center


2nd Floor, IAPL House, 19, Pusa Road, Karol Bagh, New Delhi – 110005 | helpdesk@forumias.academy| 9821711605
Page 30 of 48

PTS 2021| All India Open Simulator Test 0 - Solutions | ForumIAS

accept demand deposits (up to Rs 1 lakh), offer remittance services, mobile payments/transfers/purchases and
other banking services like ATM/debit cards, net banking and third-party fund transfers.
The main objective of payments bank is to widen the spread of payment and financial services to small business,
low-income households, migrant labour workforce in secured technology-driven environment
Source: Reserve Bank of India - Database (rbi.org.in) What is Payments Banks? Definition of Payments Banks,
Payments Banks Meaning - The Economic Times (indiatimes.com)
Q.60)
Ans) a
Exp) Option a is correct.
Statement 1 is incorrect. Strategic disinvestment involves sale of substantial portion of Government
shareholding in identified CPSEs upto 50 per cent or more, along with transfer of management control.
Statement 2 is correct. The policy on disinvestment has evolved considerably through President's address to
Joint Sessions of Parliament and statement of the Finance Minister's in their Budget Speeches. Strategic
disinvestment is undertaken through a consultation process among different Ministries or Departments,
including NITI Aayog.
Statement 3 is incorrect. NITI Aayog and not DIPAM to identify CPSEs for strategic disinvestment and
advice on the mode of sale, percentage of shares to be sold of the CPSE and method for valuation of the CPSE.
The Department of Investment and Public Asset Management (DIPAM) is to identify CPSEs in consultation
with respective administrative Ministries and submit proposal to Government in cases requiring Offer for Sale
of Government equity in Disinvestment through minority stake sale.
Source: DIPAM | Disinvestment Policy

Q.61)
Ans) d
Exp) Option d is correct.
Option d is correct. National Infrastructure Pipeline (NIP) encompasses an outlay of Rs 100 lakh Crore for
infrastructure projects over 5 years from 2020-2025. It is a first-of-its-kind initiative to provide world-class
infrastructure across the country and is aimed at improving the quality of life for all citizens. The project
covers both economic and social infrastructure projects. The emphasis would be on ease of living: safe drinking
water, access to clean and affordable energy, healthcare for all, modern railway stations, airports, bus
terminals and world-class educational institutes. It will create jobs, improve ease of living, and provide
equitable access to infrastructure for all, thereby making growth more inclusive.Knowledge Base:
The Government had created a task force, headed by Atanu Chakraborty, to draw up the National
Infrastructure Pipeline (NIP) for each of the years from financial years 2019-20 to 2024-25. It submitted its
final report to the Finance Minister in May 2020.
Important recommendations:
1) ₹111 lakh crore investment needed over 2020-2025 to build infrastructure projects and drive economic
growth.
2) Energy, roads, railways and urban projects are estimated to account for the bulk of projects (around 70%).
3) Aggressive push towards asset sales.
4) Monetisation of infrastructure assets.
5) Setting up of development finance institutions.

ForumIAS Guidance Center


2nd Floor, IAPL House, 19, Pusa Road, Karol Bagh, New Delhi – 110005 | helpdesk@forumias.academy| 9821711605
Page 31 of 48

PTS 2021| All India Open Simulator Test 0 - Solutions | ForumIAS

6) Strengthening the municipal bond market.


The task force has recommended setting up of three committees to monitor NIP progress and eliminate
delays, to follow up on the implementation process and to raise financial resources for the NIP.
Source: https://pib.gov.in/Pressreleaseshare.aspx?PRID=1693892
https://www.thehindu.com/business/Economy/fm-reviews-nip-as-part-of-infra-spending-
push/article33513765.ece
Q.62)
Ans) a
Exp) Option a is correct
Statement 1 is correct. Ricardian equivalence is an economic theory developed by David Ricardo in the early
19th century. It says that consumers are forward-looking and will base their spending not only on their
current income but also on their expected future income. The financing of government spending out of
current taxes or future taxes (and current deficits) will have equivalent effects on the overall economy. This
means that attempts to stimulate an economy by increasing debt-financed government spending will not be
effective because investors and consumers understand that the debt will eventually have to be paid for in the
form of future taxes.
Statement 2 is incorrect. It is called ‘equivalence’ because it argues that taxation and borrowing are
equivalent means of financing expenditure. The theory argues that people will save based on their expectation
of increased future taxes to be levied in order to pay off the debt, and that this will offset the increase in
aggregate demand from the increased government spending. This also implies that Keynesian fiscal policy will
generally be ineffective at boosting economic output and growth.
Source: Economic Survey 2020-21, Volume 1, Chapter 2, Pg. 64-65

Q.63)
Ans) c
Exp) Option c is correct.
Recently, Government of India has amended the Essential Commodities Act of 1955 and inserted a new sub-
section (1A) in Section 3 of the Essential Commodities (EC) Act, 1955
Statement 1 is incorrect. The amendment provides that foodstuffs, including cereals, pulses, potato, onions,
edible oil seeds and oils shall only be regulated under extra ordinary circumstances which may include war,
famine, extra ordinary price rise and natural calamity of grave nature.
Statement 2 is correct. According to the amendment, any action on imposing stock limit would be based on
price trigger such as hundred per cent increase in retail price of horticultural produce and fifty percent
increase in retail price of non-perishable agricultural foodstuff over the immediately preceding twelve months
or average retail price of last five years whichever is lower.
Statement 3 is correct. The Amendment includes a definition of value chain participants covering all the
entities who add value to each stage from production to consumption. It is a visionary step, one which would
radically alter the incomes and growth prospects of farmers and incentivize investment in the entire agri-sector.
Source: Economic Survey Volume 2 Pg no 215 of pdf

Q.64)
Ans) a

ForumIAS Guidance Center


2nd Floor, IAPL House, 19, Pusa Road, Karol Bagh, New Delhi – 110005 | helpdesk@forumias.academy| 9821711605
Page 32 of 48

PTS 2021| All India Open Simulator Test 0 - Solutions | ForumIAS

Exp) Option a is correct


Statement 1 is incorrect. According to provisional data from the Ministry of Commerce and Industry, China
(and not USA) regained its position as India’s top trade partner in 2020. The two-way trade between India
and China stood at USD 77.7 billion for the year 2020 while India-US bilateral trade stood at USD 75.9
billion. This was mainly due to
increased import of Chinese medical supplies, preference of Chinese mobile phones and electronic gadgets
and heavy reliance on Chinese-made heavy machinery, telecom equipment and home appliances.
Statement 2 is correct. Petroleum products is the highest exporting product from India followed by
precious stones, drug formulations and biologicals, gold and other precious metals are India’s top export
items. Top import items include crude oil, petroleum products, coal, coke and briquettes.
Statement 3 is incorrect. USA followed by UAE, China and Hong Kong are largest export destinations.
India’s merchandise trade balance improved from 2009-14 to 2014-19. However, most of the improvement was
due to more than 50% decline in crude oil prices. Source: https://indianexpress.com/article/business/china-
back-as-top-india-trade-partner-even-as-relations-sour-7200747/
https://pib.gov.in/PressReleasePage.aspx?PRID=1664831

Q.65)
Ans) a
Exp) Option a is correct.
Statement 1 is incorrect. The Financial Action Task Force (FATF) was established in July 1989 by a Group
of Seven (G-7) Summit in Paris, initially to examine and develop measures to combat money laundering.
Statement 2 is correct. The countries that support terror funding and money laundering activities are put into
the list of Non-Cooperative Countries or Territories (NCCTs), also commonly known as the blacklist. It is a
list of countries that the intragovernmental organization considers non-cooperative in the global effort to
combat money laundering and the financing of terrorism.
Grey List: Countries that are considered safe haven for supporting terror funding and money laundering are
put in the FATF grey list. This inclusion serves as a warning to the country that it may enter the blacklist.
Statement 3 is incorrect. Currently, Iran and North Korea are included in the blacklist.
Source: https://www.fatf-gafi.org/

Q.66)
Ans) c
Exp) Option c is correct.
Statement 1 is incorrect. SVAMITVA (Survey of Villages and Mapping with Improvised Technology in
Village Areas) scheme is a Central Sector Scheme aimed at “providing ‘record of rights’ to village household
owners possessing houses in inhabited rural areas in villages and issuance of property cards to the property
owners. It envisages mapping the land parcels in rural inhabited areas using drone technology and
Continuously Operating Reference Station (CORS). It aims to provide an integrated property validation
solution for rural India.
Statement 2 is correct. SVAMITVA scheme is a collaborative effort of the Ministry of Panchayati Raj, State
Panchayati Raj Departments, State Revenue Departments and Survey of India.

ForumIAS Guidance Center


2nd Floor, IAPL House, 19, Pusa Road, Karol Bagh, New Delhi – 110005 | helpdesk@forumias.academy| 9821711605
Page 33 of 48

PTS 2021| All India Open Simulator Test 0 - Solutions | ForumIAS

Statement 3 is correct. The scheme will help in streamlining planning and revenue collection in rural areas
and ensuring clarity on property rights. It will enable creation of better-quality Gram Panchayat Development
Plans (GPDPs), using the maps created under this programme.
Statement 4 is incorrect. The mapping will be done across the country in a phase-wise manner over a period of
four years - from 2020 to 2024. The program is currently being implemented in six states - Haryana,
Karnataka, Madhya Pradesh, Maharashtra, Uttar Pradesh and Uttarakhand.
Statement 5 is correct. The SVAMITVA scheme aims to bring financial stability to the citizens in rural India by
enabling them to use their property as a financial asset for taking loans and other financial benefits.Source:
https://indianexpress.com/article/explained/explained-what-is-svamitva-property-card-rural-households-
6789007/

Q.67)
Ans) b
Exp) Option b is correct.
Statement 1 is incorrect. Salaries to Indian resident working in Russian Embassy in India is not included in
GDP calculation as Russian embassy in India is not a part of domestic territory of India.
Statement 2 is correct. Profits earned by a company in India, which is owned by non-residents is included in
GDP calculation of India because the income is created within the economic (domestic) territory of India.
Statement 3 is incorrect. Profits earned by a branch of IDBI bank in Germany is not included in GDP
calculation as IDBI is situated outside the domestic territory of India.
Source: Class- 12 NCERT Introductory Macroeconomics, chapter-2 National Income accounting, page-24, 25

Q.68)
Ans) c
Exp) Option c is correct.
Option c is correct. Strait of Hormuz lay between the Persian Gulf and the Gulf of Oman.
The Suez Canal is an artificial sea-level waterway in Egypt, connecting the Mediterranean Sea to the Red
Sea through the Isthmus of Suez and dividing Africa and Asia.
Kerch Strait is the only connection between the Black Sea and the Sea of Azov. Bab-el-Mandeb Strait joins
Red Sea & Gulf of Aden.
Panama Canal is the lock-type canal that connects the Atlantic and Pacific oceans through the narrow Isthmus
of Panama.
Source: https://indianexpress.com/article/explained/explained-what-has-caused-a-traffic-jam-in-the-suez-
canal-7243543/
https://indianexpress.com/article/world/suez-canal-blockage-4-of-the-biggest-trade-chokepoints-7247898/
https://www.britannica.com/place/Strait-of-Hormuz
https://www.britannica.com/place/Kerch-Strait

Q.69)
Ans) d
Exp) Option d is correct.

ForumIAS Guidance Center


2nd Floor, IAPL House, 19, Pusa Road, Karol Bagh, New Delhi – 110005 | helpdesk@forumias.academy| 9821711605
Page 34 of 48

PTS 2021| All India Open Simulator Test 0 - Solutions | ForumIAS

Option d is correct. Minimum Support Price (MSP) is a form of market intervention by the Government to
insure agricultural producers against any sharp fall in farm prices. The minimum support prices are announced
at the beginning of the sowing season for certain.
crops on the basis of the recommendations of the Commission for Agricultural Costs and Prices (CACP).
MSP is a “minimum price” for any crop that the government considers as remunerative for farmers and hence
deserving of support. The mandated crops under MSP include 14 crops of the kharif season, 6 rabi crops and 2
other commercial crops.
1) Cereals (7) - paddy, wheat, barley, jowar, bajra, maize and ragi
2) Pulses (5) - gram, arhar/tur, moong, urad and lentil
3) Oilseeds (8) - groundnut, rapeseed/mustard, toria, soyabean, sunflower seed, sesamum, safflower seed and
nigerseed
4) Raw cotton
5) Raw jute
6) Copra
7) De-husked coconut
8) Sugarcane (Fair and remunerative price)
9) Virginia flu cured (VFC) tobacco.
No MSP is provided on tea, coffee and rubber planatation.
Source: https://indianexpress.com/article/explained/explained-how-the-1-5-times-formula-for-crops-msp-
is-calculated-7075865/
https://vikaspedia.in/agriculture/market-information/minimum-support-price

Q.70)
Ans) d
Exp) Option d is correct.
Statement 1 is incorrect. Basel III guidelines were introduced in 2010 in response to the financial crisis of
2008. The guidelines aim to promote a more resilient banking system by focusing on four vital banking
parameters viz. capital, leverage, funding and liquidity. The capital adequacy ratio is to be maintained at
12.9%. In addition, banks have to maintain a capital conservation buffer of 2.5%.
Statement 2 is incorrect. Currently, the RBI applies stricter norms and not those specified under Basel III for
capital adequacy, leading banks to set aside higher capital for loans. The deadline for the implementation of
Basel-III was March 2019 in India. It was postponed to March 2020. In light of the coronavirus pandemic, the
RBI decided to defer the implementation of Basel norms by further 6 months.
Source: https://www.thehindu.com/business/Industry/yes-bank-and-bonds/article31070609.ece
https://www.bis.org/bcbs/basel3.htm#:~:text=Basel%20III%20is%20an%20internationally,and%20risk%20ma
nagement%20of%20banks.

Q.71)
Ans) a
Exp) Option a is correct
Pair 1 is correctly matched. Chaityas are Buddhist prayer halls that house a stupa. Buddhist literature
mentions several chaityas. It also describes places associated with the Buddha’s life – where he was born

ForumIAS Guidance Center


2nd Floor, IAPL House, 19, Pusa Road, Karol Bagh, New Delhi – 110005 | helpdesk@forumias.academy| 9821711605
Page 35 of 48

PTS 2021| All India Open Simulator Test 0 - Solutions | ForumIAS

(Lumbini), where he attained enlightenment (Bodh Gaya), where he gave his first sermon (Sarnath) and where
he attained nibbana (Kusinagara). Lomus rishi caves of 3rd century BCE exhibit Chaitya- prayer hall of
Buddhists.
Pair 2 is incorrectly matched. Landholders (and not wealthy men) were known as gahapatis. The Buddha’s
followers came from many social groups. They included kings, wealthy men and gahapatis, and also humbler
folk: workers, slaves and craftspeople.
Pair 3 is incorrectly matched. Mahayana is the school of Buddhism which believes in the heavenliness of
Buddha and idol worship of Buddha and Bodhisattvas. Hinayana school on the contrary does not believe in
Idol worship and tries to attain individual salvation through self-discipline and meditation.
Source: Class XII NCERT, Themes in History, Chapter-4, Pg. 92, 95

Q.72)
Ans) b
Exp) Option b is correct.
Statement 1 is incorrect. Ganges river dolphins, also called ‘susu’, live in the Ganges-Brahmaputra-Meghna
and Karnaphuli-Sangu river systems of Nepal, India, and Bangladesh. They can only live in freshwater, are
essentially blind and hunt their preys by emitting ultrasonic sounds. They are distributed across seven states in
India: Assam, Uttar Pradesh, Madhya Pradesh, Rajasthan, Bihar, Jharkhand and West Bengal.
Statement 2 is correct. Protection status of Gangetic River Dolphin:
1) Indian Wildlife (Protection), Act 1972: Schedule I.
2) International Union for the Conservation of Nature (IUCN): Endangered.
3) Convention on International Trade in Endangered Species (CITES): Appendix I
Statement 3 is correct. Vikramshila Gangetic Dolphin Sanctuary (VGDS) in Bhagalpur district, Bihar, is
India’s only sanctuary for its national aquatic animal.
Statement 4 is incorrect. Only restricted flow of water is a threat to these dolphins. They favour areas of the
river where fish are plentiful and the water current is slower. The increase in the number of barrages and
dams impede the flow of water. This makes them susceptible to inbreeding and more vulnerable to other threats
because they cannot move to new areas.
Source: https://indianexpress.com/article/india/up-video-shows-men-beating-endangered-dolphin-to-death-
7138726/

Q.73)
Ans) c
Exp) Option c is correct.
Option 1, 2 and 3 is correct. Sundarbans is a mangrove area in the delta formed by the confluence of the
Ganges, Brahmaputra, and Meghna Rivers in the Bay of Bengal. It spans from the Hooghly River in India’s state
of West Bengal to the Baleswar River in Bangladesh.
Location: It is located in the southwestern part of the delta. It constitutes over 60% of India’s total mangrove
forest area. It covers 4,200 sq. km and includes the Sundarban tiger Reserve, Sajnekhali Wildlife Sanctuary,
Lothian Wildlife Sanctuary, Haliday Wildlife Sanctuary.
Recognition: It is a World Heritage site and a Ramsar site (a wetland site designated to be of international
importance).

ForumIAS Guidance Center


2nd Floor, IAPL House, 19, Pusa Road, Karol Bagh, New Delhi – 110005 | helpdesk@forumias.academy| 9821711605
Page 36 of 48

PTS 2021| All India Open Simulator Test 0 - Solutions | ForumIAS

Climate: Tropical Monsoon


Flora:
Forest Types:
Tidal Swamp Forests, Saline Water Type Mixed Forests, Brackish Water Type Mixed Forests, Palm Swamp
Type
Endemic Flora: Sundari, Passur, Nypa
Fauna:
Endemic Fauna: Bengal tiger, Bengal monitor lizard, Salvator lizard
Others: Gangetic dolphin, estuarine crocodile, river terrapin, olive ridley turtle,
Option 4 is incorrect. Bhagwan Mahaveer Sanctuary and Mollem National Park is a protected area located in the
Western Ghats in Goa, along the eastern border with Karnataka. The park was earlier known as Mollem game
sanctuary.
Source: https://blog.forumias.com/upsc-prelims-2020-list-of-biosphere-reserves-in-india/

Q.74)
Ans) a
Exp) Option a is correct
Option a is correct. Crustaceans are invertebrates have a hard, external shell which protects their body.
Crustaceans make up a very large group of the Arthropods which include the crabs, lobsters, crayfish,
shrimp, krill, barnacles, brine shrimp, copepods, ostracods and mantis shrimp. Crustaceans are found in a
wide range of habitats - most are
free-living freshwater or marine animals, but some are terrestrial (e.g. woodlice), some are parasitic (e.g. fish
lice) and some do not move (e.g. barnacles).
Option b is incorrect. Annelids have bodies that are divided into segments. They have very well-developed
internal organs. They don’t have any limbs. E.g: earthworms, leeches, roundworms, etc.
Option c is incorrect. Most mollusks have a soft, skin-like organ covered with a hard outside shell. Some
mollusks live on land, such as the snail and slug. Other mollusks live in water, such as the oyster, mussel, clam,
squid and octopus.
Option d is incorrect. Protozoa are simple, single-celled animals. They are the smallest of all animals and are
microscopic. They do breathe, move and reproduce like multicelled animals. E.g: amoebas, Flagellates,etc.
Source: Environment by Shankar IAS, 6th Edition, Chapter-10, Pg. 133

Q.75)
Ans) d
Exp) Option d is correct.
Option d is correct. Jerdon's courser is a nocturnal bird found only in the northern part of the state of
Andhra Pradesh in peninsular India. It is a flagship species for the extremely threatened scrub jungle. The
species was considered to be extinct until it was rediscovered in 1986 and the area of rediscovery was
subsequently declared as the Sri Lankamaleswara Wildlife Sanctuary. Its habitat includes undisturbed scrub
jungle with open areas.
Source: Environment by Shankar IAS, 6th Edition, Chapter-12, Pg. 156

Q.76)

ForumIAS Guidance Center


2nd Floor, IAPL House, 19, Pusa Road, Karol Bagh, New Delhi – 110005 | helpdesk@forumias.academy| 9821711605
Page 37 of 48

PTS 2021| All India Open Simulator Test 0 - Solutions | ForumIAS

Ans) d
Exp) Option d is correct.
Option 1 is correct. Oil spills from tankers at sea or leaks from underground storage tanks on land are very
difficult to control as oil tends to spread very fast, affecting a large area in a very short time. It is one of the most
dangerous of all water pollutants. Oil spills in water can be cleaned with the help of bregoli – a by-product of
paper industry resembling saw dust, oil zapper, micro-organisms.
Option 2 is correct. Floating barriers, called containment Booms, are helpful in restricting the spread of oil
and to allow for its recovery, removal, or dispersal.
Option 3 is correct. Skimmers are devices used for physically separating spilled oil from the water’s surface.
Knowledge Base:
Other ways to control oil spill:
1) Dispersing agents are chemicals that contain surfactants, or compounds that act to break liquid substances
such as oil into small droplets. They accelerate its natural dispersion into the sea.
2) Nutrients, enzymes, or microorganisms such as Alcanivorax bacteria or Methylocella silvestris are biological
agents that increase the rate at which natural biodegradation of oil.
3) Various sorbents (e.g., straw, volcanic ash, and shavings of polyester-derived plastic) can absorb the oil
from the water.
Source: https://response.restoration.noaa.gov/oil-and-chemical-spills/oil-spills/spill-containment-
methods.html
Environment by Shankar IAS, 6th Edition, Chapter-5, Pg. 63-64

Q.77)
Ans) b
Exp) Option b is correct.
Option b is correct. Pygmy Hog is world’s smallest, rarest wild pig and specialized member of the pig family.
They are found in relatively undisturbed, tall ‘terai’ grasslands. Formerly, the species was more widely
distributed along the southern Himalayan foothills but now is restricted to only a single remnant population
in Manas Wildlife Sanctuary and its buffer reserves. The main threats are loss and degradation of grasslands,
dry-season burning, livestock grazing and afforestation of grasslands. Hunting is also a threat to the remnant
populations.
IUCN Status: Critically Endangered
Knowledge Base:
It is one of the most useful indicators of the management status of grassland habitats. The grasslands where the
pygmy hog resides are crucial for the survival of other endangered species such as Indian Rhinoceros, Swamp
Deer, Wild Buffalo, Hispid Hare, Bengal Florican and Swamp Francolin.
Source: https://www.thehindu.com/sci-tech/energy-and-environment/happily-everafter-in-a-grass-
house/article31979150.ece
Environment by Shankar IAS, 6th Edition, Chapter-12, Pg. 153

Q.78)
Ans) b
Exp) Option b is correct.

ForumIAS Guidance Center


2nd Floor, IAPL House, 19, Pusa Road, Karol Bagh, New Delhi – 110005 | helpdesk@forumias.academy| 9821711605
Page 38 of 48

PTS 2021| All India Open Simulator Test 0 - Solutions | ForumIAS

Statement 1 is correct. Central pollution control board under Ministry of environment and Forests is
responsible for setting of standards and the timeline for the implementation of the emission norms.
Statement 2 is correct. Currently no new registration of Bharat stage IV vehicles is allowed in India. Though
those bought before March 31, 2020 will be allowed to register and can run for the next 15 years.
Statement 3 is incorrect. It would help in reducing PM in diesel cars by 70% and not 100%.
Source) https://www.business-standard.com/about/what-is-bs-vi-norms
https://www.indiatoday.in/india/story/no-sale-registration-of-bs-iv-vehicles-permitted-india-supreme-
court-1689402-2020-06-16

Q.79)
Ans) d
Exp) option d is correct.
Statement 1 is incorrect. Net-zero carbon emissions refers to the situation when any emission (carbon dioxide
or other GHGs) from any source is balanced by absorbing an equivalent amount of emission from the
atmosphere. It differs from zero-carbon, which requires no carbon to be emitted.
Statement 2 is incorrect. The 2015 Paris Agreement The Paris Agreement only requires every signatory to take
the best climate action it can. It is not mandatory for all the countries to set Net zero carbon emission target .
several states have announced their “net zero carbon emission” targets in the recent past including all G-7 states
(except the US) and 11 G20 members, with mid-century (2050 or 2060) net-zero targets.
Statement 3 is incorrect. India has not yet set Net Zero Carbon Emission target but top Indian government
officials are debating whether to set a target for net-zero greenhouse gas emissions by 2050. net-zero goals do
not figure in the 2015 Paris Agreement, the new global architecture to fight climate change.
Knowledge base: Net-zero carbon Emission means reducing greenhouse gas emissions with the goal of
balancing the emissions produced and emissions removed from the earth’s atmosphere. Whereas Carbon
neutrality means balancing greenhouse gas (GHG) emissions by ‘offsetting’ – or removing from the
atmosphere – an equivalent amount of carbon for the amount produced.
Source: https://www.livemint.com/news/india/india-considers-net-zero-emissions-target-by-2050-can-it-
achieve-11616130386346.html
https://blog.forumias.com/what-is-net-zero-target-how-fair-and-realistic-these-targets-are/
https://www.herbertsmithfreehills.com/carbon-neutral-and-net-zero-carbon-whats-the-difference-and-why-
does-it-matter

Q.80)
Ans) c
Exp) Option c is correct.
Statement 1 is incorrect. Compared to vehicles powered by the internal combustion engine, fuel-cell powered
vehicles have very high energy conversion efficiency, and near-zero pollution, CO2 and water vapour being
the only emissions.
FCEVs do not need to be plugged in for charging, like battery-powered EVs.
Statement 2 is correct. Fuel cell systems are excellent candidates for small-scale decentralized power
generation. Fuel cells can supply combined heat and power to commercial buildings, hospitals, airports and

ForumIAS Guidance Center


2nd Floor, IAPL House, 19, Pusa Road, Karol Bagh, New Delhi – 110005 | helpdesk@forumias.academy| 9821711605
Page 39 of 48

PTS 2021| All India Open Simulator Test 0 - Solutions | ForumIAS

military installation at remote locations. Fuel cells have efficiency levels up to 55% as compared to 35% of
conventional power plants.
Statement 3 is incorrect. High initial cost is the biggest hurdle in the widespread commercialization of fuel
cells.
Knowledge Base:
Fuel cells are electrochemical devices that convert the chemical energy of a fuel directly and very efficiently
into electricity (DC) and heat, thus doing away with combustion. The most suitable fuel for such cells is
hydrogen or a mixture of compounds containing hydrogen. A fuel cell consists of an electrolyte sandwiched
between two electrodes. Oxygen passes over one electrode and hydrogen over the other, and they react
electrochemically to generate electricity, water, and heat.Source: Environment by Shankar IAS, 6th Edition,
Chapter-6, Pg. 90-91

Q.81)
Ans) b
Exp) Option b is correct.
The Sholas are found in the upper reaches of the Nilgiris, Anamalais, Palni hills, Kalakadu, Mundanthurai
and Kanyakumari in the states of Tamil Nadu and Kerala. They are classified as the southern mountain
temperate forests. These forests are found sheltered in valleys with sufficient moisture and proper drainage, at
an altitude of more than 1,500 metres. The upper reaches are covered with grasslands, known as Shola
grasslands.
Some of the other trees of this forest of economic significance include, magnolia, laurel, cinchona and wattle.
Such forests are also found in the Satpura and the Maikal ranges.
Knowledge Base: The Tropical moist deciduous forests are found in the regions which record rainfall between
100-200 cm. They are found in the northeastern states along the foothills of Himalayas, eastern slopes of the
Western Ghats and Orissa.
Tropical evergreen forests are found in the western slope of the Western Ghats, hills of the northeastern
region and the Andaman and Nicobar Islands. They are found in warm and humid areas with an annual
precipitation of over 200 cm.
Source: NCERT, Indian physical environment, class XI, page no. 60

Q.82)
Ans) a
Exp) Option a is correct.
Option a is incorrect. Kautilya advises the king to promulgate dharma when the social order based on the
varnas and ashrams perishes. The king is called by him dharmapravartaka, which means promulgator of the
social order.
Option b is correct. Domestic slaves were found in India from vedic times. For the first time in this period, the
slaves were engaged in agricultural work on a large scale. The state-maintained farms, on which numerous
slaves and hired labourers were employed.
Option c is correct. The Mauryan period constitutes a landmark in the system of taxation in ancient India. An
elaborate machinery for tax assessment first appears in the Mauryan period. The samaharta was the highest
officer in charge of assessment. The sannidhata was the chief custodian of the state treasury and store house.

ForumIAS Guidance Center


2nd Floor, IAPL House, 19, Pusa Road, Karol Bagh, New Delhi – 110005 | helpdesk@forumias.academy| 9821711605
Page 40 of 48

PTS 2021| All India Open Simulator Test 0 - Solutions | ForumIAS

Option d is correct. The Mauryan empire maintained a vast bureaucracy. The important functionaries were
called tirthas. Asoka appointed rajukas for the administration of justice in the empire. Asoka also appointed
dhrmamahamatras for propagating dharma among various social groups.
Source: Old NCERT, Ancient India, class XI, page no. 93, 94, 91

Q.83)
Ans) d
Exp) option d is correct.
Statement 1 is correct. Agroecology is an integrated approach that simultaneously applies ecological and social
concepts and principles to the design and management of food and agricultural systems. It seeks to optimise the
interactions between plants, animals, humans and the environment while taking into consideration the social
aspects that need to be addressed for a sustainable and fair food system.
Statement 2 is incorrect. Agroecology is fundamentally different from other approaches to sustainable
development. It is based on bottom-up and territorial processes, helping to deliver contextualised solutions to
local problems i.e. no one size fits all approach.
Statement 3 is correct. “Paramparagat Krishi Vikas Yojana” is an elaborated component of Soil Health
Management (SHM) of major project National Mission of Sustainable Agriculture (NMSA). Under PKVY
Organic farming is promoted through adoption of organic village by cluster approach and PGS certification.It is
based on the concept of agroecology.
Knowledge base: Rather than tweaking the practices of unsustainable agricultural systems, agroecology seeks to
transform food and agricultural systems, addressing the root causes of problems in an integrated way and
providing holistic and long-term solutions. This includes an explicit focus on social and economic dimensions
of food systems.
Agroecology places a strong focus on the rights of women, youth and indigenous peoples.
Source: https://blog.forumias.com/answered-what-do-you-understand-by-the-term-agroecology-how-
agroecology-is-a-better-alternative-to-other-climate-smart-agriculture-models/

Q.84)
Ans) b
Exp) Option b is correct.
Statement 1 is incorrect. Biomethanation is anaerobic digestion of organic materials which is converted into
biogas. Anaerobic digestion (AD) is a bacterial fermentation process that operates without free oxygen and
results in a biogas containing mostly methane and carbon dioxide. Biomethanation has dual benefits as it gives
biogas as well as manure as end product.
Statement 2 is correct. The biogas generated from Biomethanation process can be burned directly in a gas
boiler/burner to produce heat for thermal application industries and cooking or burnt in a gas engine to produce
electricity.
Statement 3 is correct. The biogas can be cleaned to remove the carbon dioxide and other substances, to
produce BioCNG. This can be injected into the national gas grid to be used in the same way as natural gas, or
used as a vehicle fuel.

ForumIAS Guidance Center


2nd Floor, IAPL House, 19, Pusa Road, Karol Bagh, New Delhi – 110005 | helpdesk@forumias.academy| 9821711605
Page 41 of 48

PTS 2021| All India Open Simulator Test 0 - Solutions | ForumIAS

Knowledge Base: This technology can be conveniently employed in a decentralized manner for biodegradation
of segregated organic wet wastes such as wastes from kitchens, canteens, institutions, hotels, and slaughter
houses and vegetables markets.
Source: https://mnre.gov.in/waste-to-energy/current-status

Q.85)
Ans) b
Exp) option b is correct.
option a is correct. Medicanes are extra tropical storms in the Mediterranean Sea, known as ‘Medicanes’ or
‘Mediterranean Hurricanes. On September 18, 2020, a medicane named Ianos made landfall along the coast of
Greece and caused heavy rainfall and flooding on the islands of Zakynthos, Kefalonia and Ithaca.
option b is incorrect. Medicanes occur more in colder waters than tropical cyclones, hurricanes and typhoons.
Hence, the cores of these storms are also cold, as compared to the warm cores of tropical cyclones. Warmer cores
tend to carry more moisture (hence rainfall), are bigger in size and have swifter winds.
Option c is correct. These are typically smaller in diameter and have lower wind speeds than true tropical
cyclones. Sometimes, warm-cored tropical cyclones transform into cold-cored extratropical cyclones and in rare
cases, the opposite can also happen
Option d is correct. Hazards posed by Medicanes is not usually from destructive winds but through life-
threatening torrential rains and flash floods. A La Niña produces more rain in the central eastern part, where
most of the Mediterranean cyclones develop. The slopes and the convection rising from sea waters can combine
to spin off these cyclonic storms that become a Medicane if the 10-minute average wind speeds are greater than
99 km / hr.
Source: https://www.downtoearth.org.in/news/climate-change/human-induced-climate-change-could-
increase-medicanes-say-experts-73478

Q.86)
Ans) d
Exp) Option d is correct.
Statement 1 is correct. Food Corporation of India (FCI) sells surplus stocks of wheat and rice (only) under
Open Market Sale Scheme (Domestic) at pre-determined prices through e-auction in the open market. Under
the OMSS, the rate of rice is fixed at Rs.22/kg and wheat- Rs.21/kg.
Statement 2 is correct. FCI has begun Open Market Sale Scheme (OMSS) for bulk buyers such as food
companies and flour millers. FCI godowns are overflowing with grain stock at three times the buffer stock
norms of 21.04 million tonnes, prompting the agency to start sales to bulk buyers to make space for the new
crop.
Statement 3 is correct. Under it e-auction is held is done from time to time to enhance the supply of food
grains, especially wheat during the lean season and thereby moderate the open market prices especially in the
deficit regions.
Source: https://dfpd.gov.in/omss_C.htm, https://m.economictimes.com/markets/commodities/news/excess-
buffer-stock-makes-fci-begin-open-market-sale/articleshow/69317047.cms and
https://pib.gov.in/PressReleasePage.aspx?PRID=1623561

ForumIAS Guidance Center


2nd Floor, IAPL House, 19, Pusa Road, Karol Bagh, New Delhi – 110005 | helpdesk@forumias.academy| 9821711605
Page 42 of 48

PTS 2021| All India Open Simulator Test 0 - Solutions | ForumIAS

Q.87)
Ans) a
Exp) Option a is correct.
Statement 1 is correct. India cultivates all of its coffee under a well-defined two-tier mixed shade canopy,
comprising evergreen leguminous trees. Shade trees prevent soil erosion on a sloping terrain; they enrich the
soil by recycling nutrients from deeper layers, protect the coffee plant from seasonal fluctuations in
temperature, and play host to diverse flora and fauna.
Statement 2 is correct. In India, a wide variety of spices and fruit crops like pepper, cardamom, vanilla, orange
and banana grow alongside coffee plants.
Statement 3 is incorrect. Coffee Arabica and Coffee Robusta are the two main varieties of coffee grown in
India accounting for 49 per cent and 51 per cent of area respectively under coffee.
The restricted agro-climatic conditions have forced the coffee plantations to confine themselves to small area in
south India comprising hill areas around Nilgiris. Almost the entire production is shared by three states namely
Karnataka, Kerala and Tamil Nadu.
Karnataka is the largest producer accounting for about 70 per cent of total coffee production and 60 per cent of
the area under coffee in India. This state also gives the highest yield of 10.3 quintals/hectare. Most of the 4,650
plantations are at about 1,370 metres above sea level where annual rainfall is 125-150 cm.
Source: https://www.indiacoffee.org/coffee-regions-india.html

Q.88)
Ans) b
Exp) Option b is correct.
Statement 1 is correct. Gelatin sticks are cheap explosive materials used by industries for the purpose of mining
and construction related work, like building structures, roads, rails and tunnels etc. They cannot be used without
a detonator.
Statement 2 is correct. The manufacture is regulated by Petroleum and Explosives Safety Organization
(PESO), formerly known as Department of Explosives. The PESO comes under the Ministry of Commerce and
Industry, and carries out administration work of the Explosive Act 1884 and Explosives Rules 2008 etc.
Statement 3 is incorrect. In India, prior license is required not just to make, but also to sell and buy explosives.
This includes gelatin sticks also.
Source: https://indianexpress.com/article/explained/what-are-gelatin-sticks-7251051/

Q.89)
Ans) c
Exp) Option c is correct.
Statement 1 is incorrect. In zero tillage technique, a new crop is planted in the residues of the previous crop
without any prior soil tillage or seed bed preparation. This is possible when all the weeds are controlled by the
use of herbicides.
Statement 2 is correct. Zero tilled soils are homogenous in structure with a greater number of earthworms. It
also results in increase in organic matter content due to less mineralization.
Statement 3 is incorrect. One of the important advantages of zero tillage technique is that it reduces surface
runoff due to presence of mulch.

ForumIAS Guidance Center


2nd Floor, IAPL House, 19, Pusa Road, Karol Bagh, New Delhi – 110005 | helpdesk@forumias.academy| 9821711605
Page 43 of 48

PTS 2021| All India Open Simulator Test 0 - Solutions | ForumIAS

Statement 4 is correct. The disadvantage of this technique is that it requires application of higher amount of
nitrogen for mineralization of organic matter. Sometime it also results in high number of volunteer plants and
buildup of pests.
Source: Shankar IAS Environment, 6th edition, page no. 280

Q.90)
Ans) c
Exp) Option c is correct.
Statement 1 is correct. The circular bioeconomy – conceived as an economy powered by nature and its
sustainable resources – offers a unique opportunity to use renewable natural capital to holistically transform
and manage land, food, health and industrial systems as well as cities. It focuses on minimizing waste,
replacing the wide range of non-renewable, fossil-based products currently in use in the quest to reduce
emission.
Statement 2 is incorrect. Basically to reduce pressure on natural agricultural and forestry resources beyond
the sustainable limits, Circular bio economy comes into picture.
Statement 3 is correct. National policy on Bio-fuels 2018 is the government initiative for developing a
circular bio economy vision of India.
Statement 4 is correct. Implementing the Circular Bio-economy would provide opportunities for the
Indian business systems to continue with its growth trajectory without creating undue stress on the present
resource supply.
Source: https://birac.nic.in/webcontent/1594624859_Regional_Competitiveness_of_Indian_Bioeconomy.pdf

Q.91)
Ans) c
Exp) Option c is correct.
Statement 1 is correct. It is carcinogenic to humans. In 2015, it was classified by the International Agency for
Research on Cancer (IARC) as a probable carcinogen to humans.
Statement 2 is correct. Malathion kills insects by preventing their nervous system from working properly.
So, the insects can't move or breathe normally and they die.
Statement 3 is correct. India has delivered 20,000 litres of the pesticide Malathion to Iran via the strategic
Chabahar port to help mount a coordinated regional response to the threat posed by swarms of desert
locusts.
Statement 4 is incorrect. Malathion mixes with water and can move quickly through soil. Because of these
properties, malathion can be found in surface waters such as streams, and sometimes it is found in well
water. So, it is soluble in water.
Source: http://npic.orst.edu/factsheets/malagen.html https://www.hindustantimes.com/world-news/india-
delivers-20-000-litres-of-pesticide-to-iran-to-fight-locust-swarms-101616091077019.html

Q.92)
Ans) d
Exp) Option d is correct.

ForumIAS Guidance Center


2nd Floor, IAPL House, 19, Pusa Road, Karol Bagh, New Delhi – 110005 | helpdesk@forumias.academy| 9821711605
Page 44 of 48

PTS 2021| All India Open Simulator Test 0 - Solutions | ForumIAS

Rare Earth Elements or Rare Earth Metals are a set of 17 chemical elements in the periodic table — the 15
lanthanides, plus scandium and yttrium, which tend to occur in the same ore deposits as the lanthanides, and
have similar chemical properties.
The 17 Rare Earths are cerium (Ce), dysprosium (Dy), erbium (Er), europium (Eu), gadolinium (Gd), holmium
(Ho), lanthanum (La), lutetium (Lu), neodymium (Nd), praseodymium (Pr), promethium (Pm), samarium
(Sm), scandium (Sc), terbium (Tb), thulium (Tm), ytterbium (Yb), and yttrium (Y).
One of the Rare Earths, promethium, is radioactive.
These elements are important in technologies of consumer electronics, computers and networks,
communications, clean energy, advanced transportation, healthcare, environmental mitigation, and
national defence, among others.
Scandium is used in televisions and fluorescent lamps, and yttrium is used in drugs to treat rheumatoid arthritis
and cancel.
Rare Earth elements are used in space shuttle components, jet engine turbines, and drones. Cerium, the most
abundant Rare Earth element, is essential to NASA’s Space Shuttle Programme.
Source: https://indianexpress.com/article/explained/explained-what-are-rare-earths-and-why-is-us-military-
getting-involved-in-their-processing/

Q.93)
Ans) b
Exp) Option b is correct.
Statement 1 is incorrect. Coriolis force deflects the wind to the right direction in the northern hemisphere and
to the left in the southern hemisphere. The deflection is more when the wind velocity is high. The Coriolis force
is directly proportional to the angle of latitude. It is maximum at the poles and is absent at the equator.
Statement 2 is correct. The troposphere is the lowermost layer of the atmosphere. Its average height is 13 km
and extends roughly to a height of 8 km near the poles and about 18 km at the equator. Thickness of the
troposphere is greatest at the equator because heat is transported to great heights by strong convectional
currents.
Statement 3 is incorrect. The zone separating the troposphere from stratosphere is known as the tropopause.
The air temperature at the tropopause is about minus 80 degree Celsius over the equator and about minus 45
degree Celsius over the poles. Hence it is apparently a paradox that the lowest temperature in the atmosphere
is vertically overhead the equator rather than over the poles.
Source: NCERT, fundamentals of physical geography, class XI, page no. 77, 91.
Sir, the statement - the lowest temperature in the atmosphere is vertically overhead the equator rather than
over the poles – is not there, though the explanation is given on page 91, in the new NCERT. But the statement
can be found in Old NCERT, Principles of geography, page no. 93.

Q.94)
Ans) b
Exp) Option b is correct.
Statement 1 is incorrect. India is not a net exporter of urea. India's dependency on import at present is to the
extent of 25% of our requirement of Urea, 90% in case of Phosphates, either as raw material or finished
fertilizers.

ForumIAS Guidance Center


2nd Floor, IAPL House, 19, Pusa Road, Karol Bagh, New Delhi – 110005 | helpdesk@forumias.academy| 9821711605
Page 45 of 48

PTS 2021| All India Open Simulator Test 0 - Solutions | ForumIAS

Statement 2 is correct. In the case of potash, India relies on imports to fulfil its entire consumption, bringing in
more than 4 million tonnes every year. The Government has been encouraging Indian Companies to establish
Joint Ventures abroad in Countries which are rich in fertilizer resources for production facilities with buy back
arrangements and to enter into long term agreement for supply of fertilizers and fertilizer inputs to India.
Source: https://fert.nic.in/sites/default/files/2020-09/Annual-Report-2019-20.pdf
https://www.reuters.com/article/india-potash-subsidies-idUSL3N2CA3CU

Q.95)
Ans) a
Exp) option a is correct answer.
Statement 1 is incorrect. It is being implemented by the Asian Disaster Preparedness Center (ADPC) and
Regional Integrated Multi-Hazard Early Warning System for Africa and Asia (RIMES).
Statement 2 is incorrect. The Climate Adaptation and Resilience for South Asia project, or CARE for South
Asia, is a recently launched programme that supports the region in building resilience to climate change by
improving the availability of regional data and knowledge, developing guidelines, tools and capacities, and
promoting climate-resilient decisions, policies and investments across key sectors in beneficiary countries.
Hence
Statement 3 is correct. It has been launched by the World Bank.
Knowledge Base:
Components:
Promote evidence-based climate-smart decision making
1) Expand Regional Resilience Data and Analytics Services (RDAS)
2) Strengthen national-level sectoral Decision Support Systems (DSS) for climate resilience.
3) Training for climate-informed decision making
Enhance policies, standards and capacities for climate-resilient development
1) Advisory services for policy and investment interventions
2) Promote climate-resilient design and standards
3) Implementation support to climate-risk management solutions
4) Innovation for climate adaptation and resilience
Outcomes:
1) Improved access to regional climate information and analytics for climate-informed decision making.
2) National-level planning and decision-making tools are better climate risk-informed.
3) Regional climate resilience guidelines incorporated into national standards.
4) Sectoral investments supported to include climate risks and resilient design.
5) Institutional capacities strengthened to undertake climate-informed policies and planning
Geographical Outreach:
1) South Asia Region
2) Bangladesh
3) Nepal
4) Pakistan
Sectoral Focus:
1) Climate-smart agriculture

ForumIAS Guidance Center


2nd Floor, IAPL House, 19, Pusa Road, Karol Bagh, New Delhi – 110005 | helpdesk@forumias.academy| 9821711605
Page 46 of 48

PTS 2021| All India Open Simulator Test 0 - Solutions | ForumIAS

2) Integrated water resources management


3) Resilient transport infrastructure
4) Finance
5) Policy and planning
Duration: May 2020 - August 2025
Source: https://www.careforsouthasia.info/
https://www.worldbank.org/en/news/loans-credits/2020/05/12/south-asia-climate-adaptation-and-
resilience-for-south-asia-care-project

Q.96)
Ans) b
Exp) Option b is correct.
The correct order of the places from south to north is –
1) Shipki La pass (Himachal Pradesh) – Shipki La, an ancient trade route. China says the mountain pass and its
surrounding region is on its side, but India has asserted that since it’s on the watershed the territory should
belong to it.
2) Pangong Tso lake (Ladakh) - Pangong Tso lake is site of the first clash in 2020 between India and China.
3) Galwan valley (Ladakh) - Galwan Valley, where Shyok and Galwan rivers meet, was previously the site of
clashes in 1962 and is a key strategic area.
4) Daulat Beg Oldi (DBO) (Ladakh) – It is a key northern region close to the Karakoram Pass. Construction of
the Darbuk-Shyok-DBO (DSDBO) road by India from Leh through Chushul — that leads to DBO, a
military base with an airstrip, was completed last year.
Source: https://timesofindia.indiatimes.com/india/5-point-solution-but-27-points-of-dispute-between-india-
and-china/articleshow/78075456.cms

Q.97)
Ans) b
Exp) Option b is correct.
Pair 1 is incorrectly matched. Alamgirpur is prominent archaeological site of the Indus Valley Civilization in
Uttar Pradesh’s Meerut district. Also known as Parasaram-ka-khera, this settlement along the Yamuna River
existed from 3300 BC to 1300 BC from the Harappan period. Among major discoveries made here during the
excavation were several ceramic items like roof tiles, cups, vases, beads and carts, among others. Also, a humped
bull and broken copper blade were unearthed at this site.
Pair 2 is correctly matched. Karanpura is in Rajasthan. Karanpura is an archeological site near Bhadra city of
Hanumangarh district in Rajasthan, India. It belongs with ancient Indus Valley Civilization. Harappan pottery
has been found after excavation.
Pair 3 is correctly matched. Rojdi is an archaeological site belonging to the Indus valley civilization. It is located
on the northern bank of the Bhadar River, near Rajkot in Gujarat.
Pair 4 is incorrectly matched. Rakhigarhi is in Haryana is among the largest settlements of the Indus Valley
that was discovered here. The site existed from 2600 BC to 1900 BC. During the excavation, it was found that
Rakhigarhi was an extremely well-planned city having great roads and quite urbanised sewage system. A
number of terracotta statues, bronze toys, and other artefacts were unearthed during the excavation.

ForumIAS Guidance Center


2nd Floor, IAPL House, 19, Pusa Road, Karol Bagh, New Delhi – 110005 | helpdesk@forumias.academy| 9821711605
Page 47 of 48

PTS 2021| All India Open Simulator Test 0 - Solutions | ForumIAS

Knowledge Base: Other Important Indus Valley sites in India –


Gujarat – Dholavira, Rangpur, Babar Kot, Bet Dwarka, Bhagatrav, Desalpur, Gola Dhoro, Surkotada, , Pabumath
Haryana – Balu, Banawali, Bhirrana, Farmana, Siswal
Rajasthan – Baror, Kalibangan
Uttar Pradesh – Bargaon, Hulas, SanauliSource: https://timesofindia.indiatimes.com/travel/destinations/5-
indus-valley-civilisation-archaeological-sites-to-visit-in-india/as65165890.cms

Q.98)
Ans) b
Exp) Option b is correct.
Pair 1 is incorrectly matched. Mercury compounds in waste water are converted by bacterial action into
extremely toxic methyl mercury, which can cause numbness of limbs, lips and tongue, deafness, blurring of
vision and mental derangement. A cripling deformity called Minamata disease due to consumption of fish
captured from mercury contaminated Minamata Bay in Japan was detected in 1952.
Pair 2 is correctly matched. Excess nitrate in drinking water reacts with hemoglobin to form non-functional
methaemoglobin, and impairs oxygen transport. This condition is called methaemoglobinemia or blue baby
syndrome.
Pair 3 is correctly matched. Water contaminated with cadmium can cause itai itai disease also called ouch-ouch
disease (a painful disease of bones and joints) and cancer of lungs and liver.
Pair 4 is incorrectly matched. The compounds of lead cause anaemia, headache, loss of muscle power and
bluish line around the gum.
Source:
Shankar IAS, environment, 6th edition, page no. 62

Q.99)
Ans) d
Exp) Option d is correct.
Statement 1 is incorrect. Anticyclones are opposite to cyclones. In anticyclones, the centre has the high pressure
and the isobars are far apart. The pressure gradient is gentle and winds are light.
Statement 2 is correct. Anticyclones are normally associated with fine weather. Skies are clear, the air is calm
and temperatures are high in summer but cold in winter.
Statement 3 is correct. Winds in anticyclones blow outwards and are also subject to deflection. Anticyclones
blow clockwise in the northern hemisphere and anticlockwise in the southern hemisphere.
Source: GC Leong, page no. 113

Q.100)
Ans) c
Exp) Option c is correct.
Statement 1 is incorrect. Asiatic lions are confined to Gir National Park and its surrounding environments in
Gujarat’s Saurashtra district. Kuno-Palpur Wildlife Sanctuary has been identified for the relocation of the lions
but the same has not been implemented yet.

ForumIAS Guidance Center


2nd Floor, IAPL House, 19, Pusa Road, Karol Bagh, New Delhi – 110005 | helpdesk@forumias.academy| 9821711605
Page 48 of 48

PTS 2021| All India Open Simulator Test 0 - Solutions | ForumIAS

Statement 2 is correct. Asiatic lions are included in the Schedule I of the Wildlife (Protection) Act of 1972.
They are endangered species under IUCN. They are also included in Appendix I of CITES.
Statement 3 is correct. Project Lion has been launched by the Government of India, on 15th August 2020, for
the conservation of the Asiatic Lion. It aims to focus on habitat development, engaging technologies in lion
management, addressing the issues of disease in lions, and will also be addressing the Human-Wildlife conflict.
Knowledge Base: Asiatic lions are slightly smaller than African lions. The most striking morphological
character, which is always seen in Asiatic lions, and rarely in African lions, is a longitudinal fold of skin
running along its belly.
Source: https://blog.forumias.com/project-lion-proposal-identifies-6-relocation-sites-apart-from-kuno-
palpur/
https://www.downtoearth.org.in/news/wildlife-biodiversity/project-lion-proposal-identifies-6-relocation-
sites-apart-from-kuno-palpur-73922
https://www.wwfindia.org/about_wwf/priority_species/threatened_species/asiatic_lion/

ForumIAS Guidance Center


2nd Floor, IAPL House, 19, Pusa Road, Karol Bagh, New Delhi – 110005 | helpdesk@forumias.academy| 9821711605

You might also like